OB review questions Flashcards

1
Q

643–Which of the following drugs does NOT pass the placenta easily?
A. Etomidate
B. Ephedrine
C. Atropine
D. Glycopyrrolate

A

D) glycopyrrolate

The fetal/maternal (F/M) drug ratio is a way to quantitatively describe drug transfer across the placenta. Time is also important when considering how much drug crosses into the fetus. Many anesthetic drugs cross the placenta such as local anesthetics, intravenous induction agents (e.g., propofol [F/M ratio of 0.7- 1.1], etomidate [F/M ratio of 0.5], ketamine [F/M ratio of 0.5]), inhalation agents (e.g., volatile anesthetics and nitrous oxide [F/M ratio of 0.7]), and narcotics (e.g., fentanyl [F/M ratio of 0.4], remifentanil [F/M ratio of 0.9], morphine [F/M ratio of 0.6]) and with time may affect the fetus/newborn. For vasopressors, ephedrine has an F/M ratio of 0.7, whereas phenylephrine has an F/M ratio of 0.2. The ionized neuromuscular blocking agents do not readily cross the placenta (F/M ratios of non-depolarizing drugs are around 0.1-0.2); succinylcholine, a depolarizing muscle relaxant, crosses very poorly as well. The anticholinergic drugs atropine and scopolamine have F/M drug ratios of 1.0 and readily cross the placenta, whereas glycopyrrolate has an F/M drug ratio of 0.1 and poorly crosses the placenta. Because the anticholinesterase agents (neostigmine, pyridostigmine, and edrophonium) cross the placenta to a limited extent but more so than glycopyrrolate, a pregnant patient undergoing nonobstetric surgery in which neuromuscular blocking drugs are being reversed with anticholinesterase agents should have atropine rather than glycopyrrolate used with the anticholinesterase mixture to prevent possible fetal bradycardia (Chestnut: Chestnut’s Obstetric Anesthesia, ed 5, pp 63–69; Suresh: Shnider and Levinson’s Anesthesia for Obstetrics, ed 5, pp 47–51).

How well did you know this?
1
Not at all
2
3
4
5
Perfectly
2
Q
  1. What is the P50 of fetal hemoglobin at term?
    A. 12
    B. 18
    C. 24
    D. 30
A

(B) Newborns have high hemoglobin levels around 15 to 20 g/100 mL. The term P50 denotes the blood oxygen tension (Pao2) that produces 50% saturation of erythrocyte hemoglobin. The P50 value of fetal hemoglobin is 18 mm Hg versus the adult value of 27 mm Hg. Thus, fetal hemoglobin has a higher affinity for oxygen than maternal hemoglobin (Chestnut: Chestnut’s Obstetric Anesthesia, ed 5, pp 83–84; Suresh: Shnider and Levinson’s Anesthesia for Obstetrics, ed 5, pp 26–27).

How well did you know this?
1
Not at all
2
3
4
5
Perfectly
3
Q

653–Cardiac output increases dramatically during pregnancy and delivery. The cardiac output returns to nonpregnant values by how long postpartum?
A. 12 hours
B.1day
C. 2 weeks
D. 6 months

A

(C) 2 weeks
The numerous changes that take place in the cardiovascular system during pregnancy provide for the needs of the fetus and prepare the mother for labor and delivery. During the first trimester of pregnancy, cardiac output increases by approximately 30% to 40%. At term, the cardiac output is increased 50% over nonpregnant values. This increase in cardiac output is due to an increase in stroke volume and an increase in heart rate. During labor, the cardiac output increases another 10% to 15% during the latent phase, 25% to 30% during the active phase, and 40% to 45% during the expulsive stage. Each uterine contraction increases the cardiac output by about 10% to 25%. The greatest increase in cardiac output occurs immediately after delivery of the newborn, when the cardiac output can increase to 75% above prelabor values. This final increase in cardiac output is attributed primarily to autotransfusion and increased venous return associated with uterine involution. Cardiac output falls to prelabor values within 2 days after delivery; however, it takes about 2 weeks for the cardiac output to decrease to non- pregnant values (Chestnut: Chestnut’s Obstetric Anesthesia, ed 5, pp 16–18; Suresh: Shnider and Levinson’s Anesthesia for Obstetrics, ed 5, pp 1–2).

How well did you know this?
1
Not at all
2
3
4
5
Perfectly
4
Q

655–Uterine blood flow at term pregnancy typically increases to about
A. 100 mL/min
B. 250 mL/min
C. 500 mL/min
D. 750 mL/min

A

(D) 750
Uterine blood flow increases dramatically from 50 to 100 mL/min before pregnancy to about 700 to 900 mL/min at term (i.e., >1 unit of blood per minute). From 70% to 90% of the uterine blood flow at term goes to the intervillous spaces. Uterine blood flow is related to the perfusion pressure (uterine arterial pressure minus uterine venous pressure) divided by the uterine vascular resistance. Thus, factors that decrease uterine blood flow include systemic hypotension, aortocaval compression, uterine contraction, and vasoconstriction (Chestnut: Chestnut’s Obstetric Anesthesia, ed 5, pp 40–42; Suresh: Shnider and Levinson’s Anesthesia for Obstetrics, ed 5, pp 23–24).

How well did you know this?
1
Not at all
2
3
4
5
Perfectly
5
Q

656

A
How well did you know this?
1
Not at all
2
3
4
5
Perfectly
6
Q

657–Which of the following cardiovascular parameters is decreased at term?
A. Central venous pressure
B. Pulmonary capillary wedge pressure
C. Systemic vascular resistance
D. Left ventricular end-systolic volume

A

(C) There is no change in central venous pressure, pulmonary capillary wedge pressure, pulmonary artery diastolic pressure, or left ventricular end-systolic volume. Left ventricular end-diastolic volume is increased, as is stroke volume, ejection fraction, heart rate, and cardiac output. Systemic vascular resistance is decreased about 20% (Chestnut: Chestnut’s Obstetric Anesthesia, ed 5, pp 16–19; Suresh: Shnider and Levinson’s Anesthesia for Obstetrics, ed 5, pp 1–3).

How well did you know this?
1
Not at all
2
3
4
5
Perfectly
7
Q
  1. Which of the following drugs does NOT pass the placenta easily?
    A. Etomidate
    B. Ephedrine
    C. Atropine
    D. Glycopyrrolate
A

(D)The fetal/maternal (F/M) drug ratio is a way to quantitatively describe drug transfer across the placenta. Time is also important when considering how much drug crosses into the fetus. Many anesthetic drugs cross the placenta, such as local anesthetics, IV induction agents (e.g., propofol [F/M ratio of 0.7-1.1], etomidate [F/M ratio of 0.5], ketamine [F/M ratio of 0.5]), inhalation agents (e.g., volatile anesthetics and nitrous oxide [F/M ratio of 0.7]), and narcotics (e.g., fentanyl [F/M ratio of 0.4], remifentanil [F/M ratio of 0.9], morphine [F/M ratio of 0.6]) and with time may affect the fetus/newborn. For vasopressors, ephedrine has an F/M ratio of 0.7, whereas phenylephrine has an F/M ratio of 0.2. The ionized neuromuscular blocking agents do not readily cross the placenta (F/M ratios of nondepolarizing drugs are around 0.1-0.2); succinylcholine, a depolarizing muscle relaxant, crosses very poorly as well. The anticholinergic drugs atropine and scopolamine have F/M drug ratios of 1.0 and readily cross the placenta, whereas glycopyrrolate has an F/M drug ratio of 0.1 and poorly crosses the placenta. Because the anticholinesterase agents (neostigmine, pyridostigmine, and edrophonium) cross the placenta to a limited extent but more so than glycopyrrolate, a pregnant patient undergoing nonobstetric surgery in which neuromuscular blocking drugs are being reversed with anticholinesterase agents should have atropine rather than glycopyrrolate used with the anticholinesterase mixture to prevent possible fetal bradycardia

How well did you know this?
1
Not at all
2
3
4
5
Perfectly
8
Q
  1. A 38-year-old obese patient is receiving subcutaneous low-molecular-weight heparin (LMWH) for thromboprophylaxis. Her epidural for an elective cesarean delivery was placed 14 hours after the heparin was stopped. She developed Horner syndrome on the left side 30 minutes after placement of the epidural. On physical examination, a T4 anesthetic level is noted, but aside from the Horner syndrome no other findings are revealed. The most appropriate course of action at this time would be to

A. Remove the epidural
B. Consult a neurosurgeon
C. Obtain a computed tomographic scan
D. None of the above

A

(D)LMWHs are used for both prophylaxis and treatment of arterial and venous thromboembolism. The elimination half-life of LMWH is 3 to 6 hours after subcutaneous injection in patients with normal renal function. With severe renal insufficiency, the half-life of LMWH can be up to 16 hours. At least 12 hours should elapse before performing any neuraxial techniques (e.g., placement or removal of an epidural catheter) to decrease the likelihood of a spinal hematoma forming after low-dose prophylaxis with LMWH (e.g., enoxaparin 30mg BID or 40mg once daily). If high-dose LMWH is used for therapeutic anticoagulation (e.g., enoxaparin 1mg/kg BID or 1.5mg/kg once daily), you should wait at least 24 hours to decrease the likelihood of a spinal hematoma forming. A postprocedure dose of enoxaparin should usually be given no sooner than 4 hours after epidural catheter is removed. In all cases, the benefit-risk of thrombosis and bleeding should be made. If the patient has back pain and unexpected neurologic paralysis, a workup for an epidural hematoma should be performed. This case demonstrates a benign condition in which the sympathetic nerve supply to the eye is blocked (Horner syndrome [triad of miosis, ptosis, and anhidrosis]). This occasionally develops after a lumbar epidural anesthetic, even when the highest dermatome level blocked is below T5. It may be related to the superficial anatomic location of the descending spinal sympathetic fibers that lie just below the spinal pia of the dorsolateral funiculus (which is within diffusion range of subanesthetic concentrations of local anesthetics in the cerebrospinal fluid) as well as increased sensitivity to local anesthetics during pregnancy

How well did you know this?
1
Not at all
2
3
4
5
Perfectly
9
Q
  1. What percentage of all pregnancies is affected by hypertension?

A. 3%-5%
B. 7%-10%
C. 15%
D. 20%

A

(B)Hypertension is defined as a systolic blood pressure (SBP) ≧140mm Hg or diastolic blood pressure (DBP) ≧90mm Hg on two occasions at least 4 hours apart, while the patient is at bed rest (unless antihypertensive therapy has been started); it occurs in 7% to 10% of all pregnancies worldwide. If the SBP is ≧160mm Hg or the DBP is ≧110mm Hg, the two readings can be done within a few minutes and antihypertensive medications can be started. Hypertension is a leading cause of maternal death worldwide. Hypertension during pregnancy is divided into four groups: preeclampsia-eclampsia, chronic hypertension (of any cause), chronic hypertension with superimposed preeclampsia, and gestational hypertension.

How well did you know this?
1
Not at all
2
3
4
5
Perfectly
10
Q
  1. A 16-year-old, anxious, preeclamptic patient in active labor develops back pain after the placement of an epidural for labor analgesia. The pain is severe, and the patient has more weakness of the legs than expected. The most appropriate course of action at this time would be to

A. Inject a higher concentration of a local anesthetic or add intravenous (IV) narcotics
B. Replace the epidural and use epidural narcotics to decrease the motor weakness
C. Reassure her that she will get better with delivery
D. Consult a neurosurgeon

A

(D)Epidural hematomas and epidural abscesses are quite rare. Severe back pain and/or leg weakness that is greater than expected (or the recurrence of weakness after partial recovery of a neuraxial block) are presenting symptoms of spinal cord compression. Epidural hematomas can develop within 12 hours of a neuraxial procedure, whereas epidural abscesses usually take days to develop and also present with fever and leukocytosis. These conditions need imaging (e.g., magnetic resonance imaging [MRI]) and neurosurgical consultation. Studies have shown that when spinal cord decompression occurs within 8 hours of the onset of paralysis, neurologic recovery is significantly better than after 8 hours. Although epidural hematoma formation is rare, clotting disorders and perhaps marked difficulty in placing a block could lead to epidural bleeding and hematoma formation. Because the preeclamptic patient may develop a coagulopathy, one should carefully evaluate her coagulation status before initiating a regional block. Most anesthesiologists would evaluate a platelet count in the preeclamptic patient and look for any clinical signs of unexplained bleeding before initiating a regional block. Because an epidural blood patch often is performed with 20mL of blood, the epidural hematoma that causes spinal cord compression is probably significantly greater

How well did you know this?
1
Not at all
2
3
4
5
Perfectly
11
Q
  1. Magnesium sulfate (MgSO4) is used as an anticonvulsant in patients with preeclampsia and for fetal neuroprotection and sometimes for short-term tocolysis. MgSO4 may produce any of the following effects EXCEPT

A. Sedation
B. Respiratory paralysis
C. Inhibition of acetylcholine (ACh) release at the myoneural junction
D. Hypertension when used with nifedipine

A

(D)The normal serum magnesium level is 1.5 to 2mEq/L, with a therapeutic range of 4 to 8mEq/L. Note: many laboratories report values in mg/dL (1mEq/L = 1.2mg/dL). As magnesium sulfate is administered IV, patients often note a warm feeling in the vein as well as some sedation. With increasing serum levels, loss of deep tendon reflexes occurs at 10mEq/L (12mg/dL), respiratory paralysis occurs at 15mEq/L (18mg/dL), and cardiac arrest at greater than 25mEq/L (>30mg/dL) can occur. Magnesium decreases the release of ACh at the myoneural junction and decreases the sensitivity of the motor endplate to ACh. This can produce marked potentiation of nondepolarizing muscle relaxants. The effect on depolarizing muscle relaxants is less clear, and most clinicians use standard intubating doses of succinylcholine (i.e., 1-1.5mg/kg) followed by a markedly reduced dose of a nondepolarizing relaxant if needed. Because magnesium antagonizes the effects of α-adrenergic agonists, ephedrine is usually preferred over phenylephrine if a vasopressor is needed to restore blood pressure, along with fluids, after a neuraxial blockade. When a calcium channel blocker, such as nifedipine, is administered along with magnesium, greater hypotension has resulted. The antidote for magnesium toxicity is calcium (which, if needed, should be administered slowly)

How well did you know this?
1
Not at all
2
3
4
5
Perfectly
12
Q
  1. Normal fetal heart rate (FHR) is

A. 60 to 100 beats/min
B. 90 to 130 beats/min
C. 110 to 160 beats/min
D. 150 to 200 beats/min

A

(C)Fetal monitors consist of a two-channel recorder for simultaneous recording of FHR and uterine activity. In looking at the FHR, one assesses the baseline rate, the FHR variability, and the periodic changes (accelerations or decelerations) that occur with uterine contractions. The normal FHR varies between 110 and 160beats/min. See also Answer 703

How well did you know this?
1
Not at all
2
3
4
5
Perfectly
13
Q
  1. Which of the following is the MOST likely cause of pregnancy-related deaths in the United States (2011-2013)?

A. Anesthesia complications
B. Hemorrhage
C. Cardiovascular disease
D. Hypertensive disorders of pregnancy

A

(C)Worldwide, hemorrhage (H), infection (I), and hypertensive disorders of pregnancy (preeclampsia [P]), or HIP, account for more than half of all maternal deaths. In the developed world, hypertensive disorders, infection, and hemorrhage account for about one third of maternal deaths. The rate of pregnancy-related mortality in the United States has been increasing from 7.2 deaths per 100,000 live births in 1987, to 14.5 deaths per 100,000 live births in 2000, to 17.3 deaths per 100,000 live births in 2013. The reason for the increase in deaths is unclear but may be related to more pregnant women having chronic health conditions such as hypertension, diabetes, obesity, and heart disease. The causes of pregnancy-related deaths in the United States for the years 2011 to 2013 were cardiovascular disease (15.5%), noncardiovascular disease (14.5%), infection or sepsis (12.7%), hemorrhage (11.4%), cardiomyopathy (11%), thrombotic pulmonary embolism (9.2%), hypertensive disorders of pregnancy (7.4%), cerebrovascular accidents (6.6%), amniotic fluid embolus (AFE) (5.5%), anesthesia complications (0.2%), and unknown causes (6.1%)

How well did you know this?
1
Not at all
2
3
4
5
Perfectly
14
Q
  1. Drugs useful in the treatment of uterine atony in an asthmatic patient with severe preeclampsia include

A. Oxytocin (Pitocin) only
B. Ergonovine (Ergotrate) or methylergonovine (Methergine) only
C. 15-Methyl prostaglandin F2α (PGF2α) (Carboprost, Hemabate) only
D. All of the above are safe and can be used alone or in combination with the others

A

(A)Uterine atony is a common cause of postpartum hemorrhage (2%-5% of all vaginal deliveries). Treatment consists of uterine massage, drugs, and, in some cases, tamponade balloon placement (e.g., Bakri with 300-500mL normal saline), uterine artery embolization, laparotomy with hemostatic sutures, or, in rare cases, hysterectomy. Drugs commonly used include oxytocin, ergot alkaloids (ergonovine, methylergonovine), prostaglandins (PGE2, PGF2α, 15-methyl PGF2α), and misoprostol. Oxytocin (Pitocin) is the first-line drug used for the treatment of uterine atony and may be used in patients with asthma or hypertensive disorders of pregnancy. If oxytocin is given as a large IV bolus, vasodilation and hypotension often result. Oxytocin is often given as 3 units over 30 seconds every 3 minutes for 3 doses or 30 units in 500mL of fluid over 2 hours or 10 units IM. The ergot alkaloids are associated with a high incidence of nausea and vomiting. They cause vasoconstriction, producing elevations in blood pressure, and are contraindicated in patients with hypertension (and in this case preeclampsia). The dose of Methergine is 0.2mg IM every 2 to 4 hours up to 5 doses. Ergot alkaloids have also been associated with bronchospasm (rarely) and may not be appropriate in asthmatic patients. Thus the ergot alkaloids are relatively contraindicated in patients with hypertension (such as preeclampsia), coronary artery disease, and asthma. The prostaglandin 15-methyl PGF2α (Carboprost, Hemabate) is the only prostaglandin currently approved for uterine atony in the United States and may cause significant bronchospasm in susceptible patients and is contraindicated in asthmatic patients. The dose of Hemabate is 0.25mg IM every 15 to 90 minutes up to 2mg. Other smooth muscle contraction-associated side effects of prostaglandin 15-methyl PGF2α include venoconstriction, as well as gastrointestinal (GI) muscle spasm (nausea, vomiting, and diarrhea). The prostaglandin E1 misoprostol (Cytotec) has been given (off label) for postpartum hemorrhage. Misoprostol can be given once rectally (800-1000mcg) or sublingually or orally (600-800mcg) and is used if oxytocin or ergot alkaloids are ineffective. In some cases, tranexamic acid 1000mg IV is given if blood loss is expected to be >500 to 1000mL over anticipated blood loss

How well did you know this?
1
Not at all
2
3
4
5
Perfectly
15
Q
  1. What is the P50 of fetal hemoglobin at term?

A. 12 mm Hg
B. 18 mm Hg
C. 24 mm Hg
D. 30 mm Hg

A

(B)Newborns have high hemoglobin levels around 15 to 20g/100mL. The term P50 denotes the blood oxygen tension (Pao2) that produces 50% saturation of erythrocyte hemoglobin. The P50 value of fetal hemoglobin is 18mm Hg versus the adult value of 27mm Hg. Thus fetal hemoglobin has a higher affinity for oxygen than maternal hemoglobin

How well did you know this?
1
Not at all
2
3
4
5
Perfectly
16
Q
  1. Side effects of terbutaline include all of the following EXCEPT

A. Hypertension
B. Hyperglycemia
C. Pulmonary edema
D. Hypokalemia

A

(A)Terbutaline is a β-adrenergic agonist with tocolytic properties and can be administered IV and subcutaneously, as well as orally. Side effects are similar to those of other β-adrenergic drugs and include tachycardia, hypotension, myocardial ischemia, pulmonary edema (0.3% incidence), hypoxemia (inhibition of hypoxic pulmonary vasoconstriction), hyperglycemia (30% incidence), metabolic (lactic) acidosis, hypokalemia (39% incidence and due to a shift of potassium from extracellular to intracellular space), anxiety, and nervousness. Electrocardiogram (ECG) changes with ST segment depression and T wave flattening or inversion may occur and typically resolve after stopping the β-adrenergic therapy. Whether these ECG changes reflect myocardial ischemia or hypokalemia is unclear

How well did you know this?
1
Not at all
2
3
4
5
Perfectly
17
Q
  1. Cardiac output increases dramatically during pregnancy and delivery. The cardiac output returns to nonpregnant values by how long postpartum?

A. 12 hours
B. 1 day
C. 2 weeks
D. 6 months

A

(C)The numerous changes that take place in the cardiovascular system during pregnancy provide for the needs of the fetus and prepare the mother for labor and delivery. During the first trimester of pregnancy, cardiac output increases by approximately 30% to 40%. At term, the cardiac output is increased 50% over nonpregnant values. This increase in cardiac output is due to an increase in stroke volume and an increase in heart rate. During labor, the cardiac output increases another 10% to 15% during the latent phase, 25% to 30% during the active phase, and 40% to 45% during the expulsive stage. Each uterine contraction increases the cardiac output by about 10% to 25%. The greatest increase in cardiac output occurs immediately after delivery of the newborn, when the cardiac output can increase to 75% above prelabor values. This final increase in cardiac output is attributed primarily to autotransfusion and increased venous return associated with uterine involution. Cardiac output falls to prelabor values within 2 days after delivery; however, it takes about 2 weeks for the cardiac output to decrease to nonpregnant values

How well did you know this?
1
Not at all
2
3
4
5
Perfectly
18
Q
  1. A 32-year-old parturient with a history of spinal fusion, severe asthma, and hypertension (blood pressure 180/110) is brought to the operating room wheezing. She needs an emergency cesarean section under general anesthesia for a prolapsed umbilical cord. Which of the following induction agents would be MOST appropriate for her induction?

A. Sevoflurane
B. Midazolam
C. Ketamine
D. Propofol

A

(D)Asthma occurs in about 4% to 8% of all pregnancies. Although sevoflurane is a good induction agent for asthmatic patients, a rapid-sequence IV induction with endotracheal intubation to secure the airway is preferred. Because midazolam has a slow onset of action, it is not recommended for a rapid-sequence induction. When inducing general anesthesia in an asthmatic patient, it is imperative to establish an adequate depth of anesthesia before placing an endotracheal tube. If the patient is “light,” then severe bronchospasm may occur. In patients with asthma, IV induction will work with ketamine or propofol. Ketamine is considered by many as the induction agent of choice due to its mild bronchodilator properties, but because propofol (also a good induction agent in asthmatic patients) does not stimulate the cardiovascular system as ketamine does, propofol would be preferred in this patient with hypertensive disorders of pregnancy. In patients with mild asthma who do not need the accessory muscles of respiration, regional anesthesia should be strongly considered if time permits because it would eliminate the need for endotracheal intubation. In addition, inhaled β2-adrenergic agonist (e.g., albuterol) and IV steroids may be beneficial

How well did you know this?
1
Not at all
2
3
4
5
Perfectly
19
Q
  1. Uterine blood flow at term pregnancy typically increases to about

A. 100 mL/min
B. 250 mL/min
C. 500 mL/min
D. 750 mL/min

A

(D)Uterine blood flow increases dramatically from 50 to 100mL/min before pregnancy to about 700 to 900mL/min at term (i.e., >1 unit of blood per minute). From 70% to 90% of the uterine blood flow at term goes to the intervillous spaces. Uterine blood flow is related to the perfusion pressure (uterine arterial pressure minus uterine venous pressure) divided by the uterine vascular resistance. Thus factors that decrease uterine blood flow include systemic hypotension, aortocaval compression, uterine contraction, and vasoconstriction

How well did you know this?
1
Not at all
2
3
4
5
Perfectly
20
Q
  1. Which one of the following statements is TRUE regarding human immunodeficiency virus (HIV) infected parturients?

A. Central neurologic blockade and epidural blood patches increase the chance of neurologic complications
B. Ninety percent of newborns of untreated HIV-seropositive mothers become infected in utero, during vaginal delivery, or with breastfeeding
C. The pharmacologic effects of benzodiazepines and narcotics are prolonged in patients taking protease inhibitors
D. The risk of seroconversion after percutaneous exposure to HIV-infected blood is about 5%

A

(C)Central neurologic blockade (i.e., epidural, spinal, or combined spinal-epidural), as well as epidural blood patches, appear to be safe for HIV-infected parturients. Vertical transmission from the mother to the newborn can occur in 15% to 40% when the mother is untreated. With antiretroviral therapy and elective cesarean delivery, the rate of transmission is reduced to about 1% to 2%. The risk of developing HIV after a needlestick injury with HIV-infected blood is 0.3%. (Risk of developing hepatitis B from a needlestick injury with hepatitis B infected blood is 30% and hepatitis C from a needlestick injury with hepatitic C infected blood is 2%-4%.) Patients taking protease inhibitors as part of their drug therapy have inhibition of cytochrome P-450, and both benzodiazepines, as well as narcotics, have prolonged effects

How well did you know this?
1
Not at all
2
3
4
5
Perfectly
21
Q
  1. Which of the following cardiovascular parameters is decreased at term?

A. Central venous pressure
B. Pulmonary capillary wedge pressure
C. Systemic vascular resistance
D. Left ventricular end-systolic volume

A

(C)There is no change in central venous pressure, pulmonary capillary wedge pressure, pulmonary artery diastolic pressure, or left ventricular end-systolic volume. Left ventricular end-diastolic volume is increased, as is stroke volume, ejection fraction, heart rate, and cardiac output. Systemic vascular resistance is decreased about 20%

How well did you know this?
1
Not at all
2
3
4
5
Perfectly
22
Q
  1. Which of the following signs and symptoms is NOT associated with amniotic fluid embolism (AFE)?

A. Chest pain
B. Bleeding (disseminated intravascular coagulation [DIC])
C. Pulmonary vasospasm with severe pulmonary hypertension and right heart failure
D. Left ventricular failure and pulmonary edema

A

(A)AFE is a very rare but serious complication of labor and delivery that results from the entrance of amniotic fluid and constituents of amniotic fluid into the maternal systemic circulation. About 10% of maternal deaths are caused by AFE, and two thirds of these deaths occur within 5 hours. Of those patients who survive the AFE, about 50% have significant neurologic dysfunction. For AFE to occur, the placental membranes must be ruptured, and abnormal open sinusoids at the uteroplacental site or lacerations of endocervical veins must exist. The classic triad is acute hypoxemia, hemodynamic collapse (i.e., severe hypotension), and coagulopathy without an obvious cause. More than 80% of these women develop cardiopulmonary arrest. Hemodynamic monitoring often shows a biphasic response; initially pulmonary vasospasm with severe pulmonary hypertension and right heart dysfunction is seen, followed by left ventricular failure and pulmonary edema. DIC occurs in about 66% of cases, and seizures occur about 50% of the time. Recently AFE is believed to be a bit different from a pure embolic event, because findings of anaphylaxis and septic shock also are involved. Bronchospasm, however, is rare (<15%) during an AFE, and chest pain is very rare (2% of patients)

How well did you know this?
1
Not at all
2
3
4
5
Perfectly
23
Q
  1. When is the fetus most susceptible to the effects of teratogenic agents?

A. 1 to 2 weeks of gestation
B. 3 to 8 weeks of gestation
C. 9 to 14 weeks of gestation
D. 15 to 20 weeks of gestation

A

(B)Organogenesis mainly occurs between the 15th and 56th days (3-8 weeks) of gestation in humans and is the time during which the fetus is most susceptible to teratogenic agents. Although all commonly used anesthetic drugs are teratogenic in some animal species, there is no conclusive evidence to implicate any currently used local anesthetics, IV induction agents, or volatile anesthetic agents in the causation of human congenital anomalies

How well did you know this?
1
Not at all
2
3
4
5
Perfectly
24
Q
  1. A 28-week estimated gestational age (EGA), 1000-g male infant is born to a 24-year-old mother who is addicted to heroin. The mother admits taking an extra “hit” of heroin before coming to the hospital because she was nervous. The infant’s respiratory depression would be best managed by

A. 0.1 mg/kg naloxone intramuscularly (IM) in the newborn’s thigh muscle
B. 0.1 mg/kg naloxone down the endotracheal tube
C. 0.4 mg naloxone IM to the mother during the second stage of labor
D. None of the above

A

(D)Opioid use during pregnancy has escalated dramatically in recent years and parallels the opioid epidemic observed in the general population. Opioid abuse during pregnancy is estimated to occur in about 5% of patients in the United States, most often with the nonprescription use of pain-relieving drugs such as oxycodone. Other opioids include morphine, heroin, methadone, meperidine, and fentanyl. The problems associated with abuse are many and include the drug effect itself and the effects of substances mixed with the narcotics (e.g., talc, cornstarch), as well as infection and malnutrition. Neonatal abstinence syndrome (NAS) or drug withdrawal syndrome has increased from 1.5 cases per 1000 hospital births in 1999 to 6 cases per 1000 hospital births in 2013. NAS is manifested by central nervous system (CNS), GI symptoms of irritability, high-pitched cry, and poor sleep and sucking reflexes that lead to poor feeding. After delivery, respiratory depression as manifested by a low respiratory rate is treated with controlled ventilation but not with naloxone. Naloxone can precipitate an acute withdrawal reaction and should not be administered to patients with chronic narcotic use (mother or newborn). The dose of naloxone to treat narcotic-induced respiratory depression in the nonaddicted newborn was 0.1mg/kg, but more recent data suggest that it may worsen the neurologic damage caused by asphyxia. Animal studies have also raised the question of complications such as pulmonary edema and cardiac arrest, as well as seizures, and current recommendations are to avoid naloxone use in the newborn. Current recommendations are to assist ventilation until the narcotic effects wear off and not to use naloxone (this includes nonaddicted mothers who have just received narcotics during labor)

How well did you know this?
1
Not at all
2
3
4
5
Perfectly
25
Q
  1. Cardiac output is GREATEST

A. During the first trimester of pregnancy
B. During the third trimester of pregnancy
C. During labor
D. Immediately after delivery of the newborn

A

(D)Immediately after delivery, the cardiac output can increase 75% above prelabor values. This is thought to result from autotransfusion and increased venous return to the heart associated with involution of the uterus, as well as increased blood return as the result of the lithotomy position. See also Answer 653

How well did you know this?
1
Not at all
2
3
4
5
Perfectly
26
Q
  1. A 1000-g, 27-week EGA boy is born with a heart rate of 80 beats/min. He has slow irregular respiratory efforts, grimaces when a suction catheter is inserted into the mouth and nose for suctioning, and flexes his limbs some but is totally cyanotic. The umbilical cord has only two vessels. The 1-minute Apgar score would be

A. 3
B. 4
C. 6
D. 7

A

(B)The Apgar score is a subjective scoring system used to evaluate the newborn and is commonly performed 1 minute and 5 minutes after delivery. If the score is less than 7, the scoring is also performed at 10, 15, and 20 minutes after delivery. A value of 0, 1, or 2 is given to each of five signs (heart rate, respiratory effort, reflex irritability, muscle tone, and color) and totaled. In this case the child gets 1 point for heart rate, 1 point for respiratory effort, 1 point for reflex irritability, 1 point for muscle tone, and 0 points for color.

How well did you know this?
1
Not at all
2
3
4
5
Perfectly
27
Q
  1. Which of the following respiratory parameters is NOT increased in the parturient?

A. Minute ventilation (MV)
B. Tidal volume (Vt)
C. Arterial Pao2
D. Serum bicarbonate

A

(D)The respiratory system undergoes many important changes during pregnancy. Oxygen consumption increases about 20% to 60%. To help supply the needed oxygen for the metabolically active mother and fetus, MV increases about 45% to 50%. The increase in MV is primarily due to an increase in Vt of 40% to 45%, with a slight increase in respiratory rate. The increase in MV produces a fall in the Paco2 to approximately 30 to 32mm Hg, and a respiratory alkalosis develops. To help get the pH back to normal, the serum bicarbonate level falls an average of 4mEq/L. The arterial Pao2 increases slightly due to the fall in Paco2

How well did you know this?
1
Not at all
2
3
4
5
Perfectly
28
Q
  1. Which of the following drugs should NOT be used during transvaginal oocyte retrieval (TVOR) for assisted reproductive technology (ART)?

A. Propofol
B. Ketamine
C. Midazolam
D. All are safe and can be used

A

(D)About 11% of women (age 15-44) have received medical evaluation and treatment for infertility at some time in their lives, with another 6% of married women (age 15-44) unable to get pregnant after 12 months of trying to conceive. In 2015 there were 231,936 ART cycles in the United States (including 4003 cycles using frozen eggs and 45,779 cycles started with the intent of freezing and storing eggs or embryos for potential future use). With fresh nondonor ART cycles, 29% resulted in a pregnancy and 24% resulted in a live birth. The oocytes can be retrieved by laparoscopy or, more commonly now, by the transvaginal oocyte retrieval (TVOR) method. Most anesthetic drugs have been studied and found not to be a problem, including propofol, midazolam, ketamine, alfentanil, fentanyl, remifentanil, and meperidine. When general anesthesia was used (laparoscopic retrieval), isoflurane with and without nitrous oxide was usually used and appeared safe. However, with increased time during general anesthesia, the oocytes retrieved earlier had better fertilization rates than the oocytes obtained near the end of the laparoscopy. It is unclear whether this was due to the anesthetics or to the lowered pH as a result of the carbon dioxide pneumoperitoneum. Etomidate has not been widely used, and patient numbers are too small to recommend its use. When morphine is used in high doses in animal studies, chromosomal abnormalities are very common (25%-33%), and morphine is not recommended for ART procedures. It is recommended to avoid using the dopamine antagonists (e.g., droperidol and metoclopramide) during ART cycles because these drugs induce hyperprolactinemia, which impairs ovarian follicular maturation. A single dose immediately before oocyte retrieval probably is safe. The 5-hydroxytryptamine type 3 (5-HT3) receptor antagonists (e.g., ondansetron, granisetron) are commonly used as antiemetics, but there is insufficient evidence to recommend their use during ART procedures. The phenothiazines and the antihistamine H1-receptor antagonists are thought to be preferred because they have been studied without adverse effects

How well did you know this?
1
Not at all
2
3
4
5
Perfectly
29
Q
  1. Which of the following conditions is associated with increased bleeding during pregnancy?

A. Lupus anticoagulant
B. Factor V Leiden mutation
C. Protein C deficiency
D. None of the above

A

(D)All of the conditions listed in this question, as well as deficiencies of antithrombin III and protein S (a cofactor for protein C), lead to hypercoagulable states. Unless treated with anticoagulation therapy, these conditions will have an increased frequency of thrombosis. These conditions may also cause placental thrombosis and insufficiency, and can increase the incidence of obstetric conditions, such as intrauterine growth restriction, preeclampsia, placental abruption, and intrauterine death. Lupus anticoagulant, also called lupus antibody, is a prothrombotic agent. It gets its name because the presence of these antibodies causes an increase in the activated partial thromboplastin (aPTT) test, as these antibodies interfere with phospholipids used to induce in vitro coagulation. However, in vivo these antibodies interact with platelet membrane phospholipids, increasing adhesions and the aggregation of platelets. Factor V Leiden mutation allows factor V to persist longer in the circulation (not metabolized as rapidly by activated protein C), leading to a hypercoagulable state. Protein C inhibits activated clotting factors V and VIII; thus, during a deficiency state, factors V and VIII persist longer in the circulation, leading again to a hypercoagulable state. During pregnancy, the incidence of thrombosis with protein C deficiency is about 25% unless anticoagulation therapy is administered

How well did you know this?
1
Not at all
2
3
4
5
Perfectly
30
Q
  1. What is the BEST way to prevent autonomic hyperreflexia in a quadriplegic woman who is to undergo induction of labor? The complete spinal cord lesion occurred 2 years ago.

A. Only IV drugs should be used; spinal and epidural anesthesia are contraindicated
B. Spinal or epidural lumbar local anesthetics such as bupivacaine alone are effective
C. Spinal or epidural narcotics such as fentanyl alone are effective
D. Autonomic hyperreflexia appears only when the complete spinal cord lesion is below T6, so there is no need to worry

A

(B)Patients with complete spinal cord lesions above T10 do not have pain with labor. However, about 85% of women with complete spinal cord lesions at the T6 and higher level will develop autonomic hyperreflexia (severe headache, hypertension, bradycardia, sweating above the lesion, and facial flushing) during labor and delivery. Autonomic hyperreflexia typically occurs with the contractions and disappears between contractions. An epidural or a spinal with local anesthetics works well to prevent and/or treat autonomic hyperreflexia. Epidural narcotics such as fentanyl alone are not effective (unless the narcotic is meperidine, which has local anesthetic properties in addition to narcotic effects). To check whether the epidural or spinal that is loaded with a local anesthetic is working in a quadriplegic patient, check the reflexes below the expected level of anesthesia (e.g., patellar) before and after the block. If the patellar reflex is present before but not after the block is performed, the block is effective. The local anesthetic concentration needed for labor epidurals (alone without narcotics) typically is 0.25% or higher. If a cesarean section is needed, 2% lidocaine with epinephrine (1:200,000) has been reported to be safe. If a cesarean section is needed with general anesthesia, typical IV anesthetics and inhalation drugs are used except for muscle relaxation, where succinylcholine is contraindicated (hyperkalemic response) and a nondepolarizing muscle relaxant such as rocuronium is preferred

How well did you know this?
1
Not at all
2
3
4
5
Perfectly
31
Q
  1. A 24-year-old gravida 2, para 1 parturient is anesthetized for emergency cesarean section. On emergence from general anesthesia, the endotracheal tube is removed and the patient becomes cyanotic. Oxygen is administered by positive-pressure bag and mask ventilation. High airway pressures are necessary to ventilate the patient, and wheezing is noted over both lung fields, along with hypoxemia. The patient’s blood pressure falls from 120/80 to 90/60 mm Hg, and heart rate increases from 105 to 150 beats/min. The MOST likely cause of these manifestations is

A. Amniotic fluid embolus (AFE)
B. Mucus plug in trachea
C. Tension pneumothorax
D. Aspiration

A

(D)The signs presented in this case—bronchospasm, high airway pressures, hypoxemia, and wheezing, followed by hypotension and tachycardia—make gastric acid aspiration the most likely cause. It is important to note that aspiration can develop not only on induction but also on extubation, as in this case. That is why it is so important always to empty the patient’s stomach with an orogastric tube after an endotracheal tube is placed in any pregnant patient over 20 weeks’ gestation undergoing general anesthesia, and to extubate the patient when she is fully awake and responsive. Morbidity and mortality occurring after gastric acid aspiration are determined by both the amount and the pH of the aspirated gastric material. Based on an animal study in which 0.4mL/kg with a pH less than 2.5 injected into the right mainstem of one rhesus monkey caused death, many have used that definition (0.4mL/kg with a pH <2.5) to categorize patients who are “at risk” for significant aspiration morbidity and mortality. Using these values, up to 70% of women who fasted before elective cesarean section are “at risk for aspiration.” Recently, it has been noted that the volume needed to cause aspiration in primates should be greater (e.g., 0.8mL/kg) and the pH less than 2.5. Regardless of the definition of the “patient at risk,” when aspiration occurs, it can be lethal. Other signs and symptoms of aspiration include sudden coughing or laryngospasm, dyspnea, tachypnea, the presence of foreign material in the mouth or posterior pharynx, chest wall retraction, cyanosis not relieved by oxygen supplementation, tachycardia, hypotension, and the development of pinky frothy exudates. The onset of these signs and symptoms is usually rapid. Early treatment consists of supplemental oxygen with positive-pressure ventilation, PEEP, or continuous positive airway pressure, and suctioning of the airway can decrease the incidence of mortality from acid aspiration. Mortality seems to be reduced when protective ventilatory strategies are used (i.e., tidal volumes of 6mL/kg with plateau pressures of <30cm H2O are better than if 12mL/kg and plateau pressures of 50cm H2O are used). Conservative compared with liberal fluid management (guided by central venous pressures and/or pulmonary artery wedge pressures) also appears to improve lung function. The use of prophylactic antibiotics and/or steroids has not been helpful. With an AFE, high airway pressures and bronchospasm are not seen, but cardiovascular collapse (including >80% cardiac arrest), DIC (60%), and seizures (>50%) are present. A mucus plug of an endotracheal tube can be associated with high airway pressures and mainly airway issues and would be extremely rare after endotracheal extubation. A tension pneumothorax would be more common during the anesthetic with positive-pressure ventilation and would most likely lead to decreased breath sounds on one side and a deviated trachea; it would not have presenting signs after endotracheal extubation. See Question 658

How well did you know this?
1
Not at all
2
3
4
5
Perfectly
32
Q
  1. A 29-year-old gravida 1, para 0 woman at 8 weeks of gestation is to undergo an emergency appendectomy under general anesthesia with isoflurane, N2O, and oxygen. Which of the following is a proven untoward consequence of general anesthesia in the unborn fetus?

A. Congenital heart disease
B. Cleft palate
C. Behavioral defects
D. None of the above

A

(D)The primary objectives in the anesthetic management of a pregnant woman undergoing general anesthesia for nonobstetric surgery are as follows: to (1) ensure maternal safety; (2) avoid teratogenic drugs; (3) avoid intrauterine fetal asphyxia; and (4) prevent the induction of preterm labor. Premature onset of labor is the most common complication associated with surgery during the second trimester of pregnancy. Performance of intra-abdominal procedures in which the uterus is manipulated is the most significant factor in causing premature labor in these patients. Neurosurgical, orthopedic, thoracic, or other surgical procedures that do not involve manipulation of the uterus do not cause preterm labor. No anesthetic agent or technique has been found to be significantly associated with a higher or lower incidence of preterm labor. Furthermore, there is no evidence that the risk of developing any of the conditions listed in this question is increased for the offspring of patients who receive general anesthesia during pregnancy

How well did you know this?
1
Not at all
2
3
4
5
Perfectly
33
Q
  1. A lumbar epidural is placed in a 24-year-old gravida 1, para 0 parturient with myasthenia gravis (MG) for labor. Select the TRUE statement regarding neonatal MG.

A. The newborn is almost always affected with myasthenia
B. The newborn is affected by maternal immunoglobulin M (IgM) antibodies
C. The newborn may require anticholinesterase therapy for up to 4 weeks
D. The newborn will need lifelong treatment

A

(C)MG is an autoimmune neuromuscular disease in which immunoglobulin G (IgG) antibodies are directed against the ACh receptors in skeletal muscle, causing patients to present with general muscle weakness and easy fatigability. Smooth muscle and cardiac muscle are not affected. About 10% to 20% of newborns born to mothers with MG are transiently affected because the IgG antibody is transferred through the placenta. Neonatal MG is characterized by muscle weakness (e.g., hypotonia, respiratory difficulty) and may appear within the first 4 days of life (80% appear within the first 24 hours). Anticholinesterase therapy may be required for several weeks, until the maternal IgG antibodies are metabolized

How well did you know this?
1
Not at all
2
3
4
5
Perfectly
34
Q
  1. A patient having which of the following conditions is LEAST likely to develop DIC?

A. Severe preeclampsia
B. Placental abruption
C. Placenta previa (bleeding)
D. Dead fetus syndrome

A

(C)Disseminated intravascular coagulation (DIC) is an acquired coagulopathy characterized by excessive fibrin deposition, depression of the normal coagulation inhibition mechanism, and impaired fibrin degradation. The formation of clots causes a depletion of platelets and factors. Laboratory diagnosis of DIC is based on the demonstration of abnormalities in platelet count (i.e., <100,000/mm3), prolonged prothrombin time (i.e., >3 seconds above normal), presence of fibrin degradation products, and fibrinogen level (i.e., ≤1g/L). DIC is associated with the following obstetric conditions: placental abruption, dead fetus syndrome, AFE, gram-negative sepsis, and severe preeclampsia. Placental abruption is the most common cause of DIC in pregnant patients. If one looks at severe placental abruptions (in which the abruption is large enough to cause fetal death), about 30% of patients will develop DIC within 8 hours of the abruption. Nonobstetric causes of DIC include sepsis and malignancy. Patients with placenta previa who are bleeding do not develop DIC because the blood loss does not induce a coagulopathy

How well did you know this?
1
Not at all
2
3
4
5
Perfectly
35
Q
  1. A 28-year-old gravida 1, para 0 parturient with Eisenmenger syndrome (pulmonary hypertension with an intracardiac right-to-left or bidirectional shunt) is to undergo placement of a lumbar epidural for analgesia during labor. It may be wise to avoid a local anesthetic with epinephrine in this patient because it

A. Lowers pulmonary vascular resistance
B. Lowers systemic vascular resistance
C. Increases heart rate
D. Causes excessive increases in systolic blood pressure (SBP)

A

(B)Eisenmenger syndrome may develop in patients with uncorrected left-to-right intracardiac shunting such as for ventricular septal defect (VSD), atrial septal defect (ASD), or patent ductus arteriosus (PDA). About half of the patients with an unrestricted and unrepaired VSD will ultimately develop Eisenmenger syndrome. In this syndrome, the pulmonary and vascular tone and right ventricular muscle undergo changes in response to the increased blood flow from the left-right shunt, producing severe pulmonary hypertension and eventually a change in the direction of the shunt to a right-to-left or bidirectional type with peripheral cyanosis. The maternal mortality rate is 30% to 50%. When the Eisenmenger syndrome develops, the pulmonary vascular resistance becomes fixed, making this condition not amenable to surgical correction. Survival beyond age 40 years is uncommon. Any event or drug that increases pulmonary vascular resistance (e.g., hypercarbia, acidosis, hypoxia) or decreases systemic vascular resistance will increase the right-to-left shunt, will exacerbate peripheral cyanosis, and may precipitate right ventricular heart failure in these patients. Controversy exists regarding pain management for these patients because pain can elevate pulmonary artery pressures and cause more shunting. Many practitioners prefer a narcotic-based analgesic (spinal or epidural). Because these patients are very dependent on preload and afterload, invasive monitors to monitor intravascular volume (e.g., central venous pressure and arterial catheter) and a pulse oximeter to evaluate the amount of shunting (e.g., a decrease in oxygen saturation may indicate an increase in right-to-left shunting) are helpful to assess the need for aggressive treatment of any fall in preload or peripheral vascular resistance. It should be recalled that centrally administered local anesthetics reduce both preload and afterload. An epidural anesthetic with a slower onset of action may be preferred to a spinal anesthetic with the faster onset of action. Low-dose epinephrine used in an epidural anesthetic can be used to decrease the absorption of local anesthetics but should be used cautiously, if at all, because a further decrease in systemic vascular resistance may result from the β effect of absorbed epinephrine, and an intravascular injection may further elevate pulmonary pressures, exacerbating the right-to-left shunt

How well did you know this?
1
Not at all
2
3
4
5
Perfectly
36
Q
  1. Which of the following patients is MOST likely to need an emergency hysterectomy for uncontrolled bleeding at the time of delivery?

A. Patient undergoing cesarean section after an unsuccessful trial of labor after cesarean (TOLAC)
B. Patient with quadruplets
C. Patient with a placenta previa (not bleeding) for an elective repeat cesarean section
D. Patient with an abdominal pregnancy

A

(C)The need for a hysterectomy for a planned repeat cesarean delivery is 0.3%, for a successful vaginal birth after cesarean is 0.1%, and for an unsuccessful TOLAC is 0.5%. With multiple gestations, uterine atony is common, and the need for a hysterectomy is sixfold in a normal delivery. However, the patient with placenta previa and a previous scar in the uterus has the highest chance of needing an emergency hysterectomy for uncontrolled bleeding at the time of delivery because of the associated placenta accreta (abnormally adherent placenta). The incidence of placenta accreta in a patient with placenta previa and no previous cesarean section is 3% to 4%, with one previous cesarean section is about 10% to 25%, and with two or more previous cesarean sections is 40% to over 60%. Most patients with placenta accreta will require a cesarean hysterectomy. The average blood loss during an emergency cesarean delivery and needed hysterectomy is 5 to 7 units of blood. Patients with a known placenta accreta should undergo a scheduled preterm cesarean delivery and hysterectomy with the placenta left in situ because any attempt to remove the placenta would likely lead to hemorrhage. Attempts to preserve the uterus with a placenta accreta has been described in select cases when hemodynamic stability is present and there is a very strong desire for a future pregnancy. This may include leaving the placenta in situ for placental involution or for only partial placenta accreta management by curettage and oversewing. With an advanced abdominal pregnancy, a laparotomy with delivery of the fetus is performed. Management of the placenta is controversial: Removal of the placenta can lead to massive hemorrhage, whereas leaving the placenta in situ is associated with an increase in infectious morbidity; however, a hysterectomy is not needed because the fetus is extrauterine

37
Q
  1. The MOST common injury recorded in the American Society of Anesthesiologists’ (ASA’s) Closed Claims Project regarding obstetric anesthetic claims is

A. Pain during anesthesia
B. Maternal nerve damage
C. Headache
D. Aspiration pneumonitis

A

(B)According to the ASA’s Closed Claims Project for Obstetric Anesthesia Claims (640 claims as of December 2010 report), maternal nerve damage (19%), neonatal brain damage (16%), and maternal death (15%) were the three most frequent claims. Other causes include headache (11%), back pain (10%), neonatal death (9%), emotional distress (8%), maternal brain damage (7%), pain during anesthesia (6%), and aspiration pneumonitis (1%)

38
Q
  1. Which of the following statements about chorioamnionitis is FALSE?

A. Chorioamnionitis occurs in about 1% of all pregnancies
B. Clinical signs include temperature higher than 38° C, maternal and fetal tachycardia, and uterine tenderness
C. Antibiotics are administered only after delivery, because intrapartum antibiotics may “obscure the results of neonatal blood cultures”
D. Epidural anesthesia can be safely administered

A

(C)Chorioamnionitis occurs in about 1% of all pregnancies. It includes the clinical signs and symptoms of infection, temperature higher than 38° C, maternal and fetal tachycardia, uterine tenderness (about 10% of patients), and/or foul-smelling amniotic fluid. Prompt delivery is the cornerstone of therapy. At one time it was thought that antibiotics should be administered only after delivery because antepartum or intrapartum antibiotics may “obscure the results of neonatal blood cultures.” However, early antepartum treatment with antibiotics leads to a decrease in maternal and neonatal morbidity, compared with delaying the antibiotics until after delivery, and is currently recommended. Epidural anesthesia has been shown to be commonly used and safe in these patients, preferably after antibiotics have been started. It seems prudent, however, to always individualize care and to weigh the risks versus the benefits of epidural anesthesia in a patient with suspected bacteremia

39
Q
  1. Which of the following statements regarding newborns with thick meconium-stained amniotic fluid is TRUE?

A. Only oral or nasal suctioning with a bulb syringe is needed in newborns that are vigorous
B. Intubation is required for all such newborns
C. Antibiotics and steroids are often needed to treat the infection
D. Respiratory distress syndrome (RDS) is common

A

(A)Meconium-stained amniotic fluid occurs in about 5% to 15% of all deliveries. Although intrapartum oropharyngeal and nasopharyngeal suction for all newborns born to mothers with meconium staining has been routine care for many years, current evidence shows no real benefit, and it is no longer recommended. In newborns who are vigorous (i.e., strong respiratory efforts, good muscle tone, and heart rate >100beats/min), no treatment other than gentle bulb syringe suctioning to clear oral or nasal secretions is needed. Intubation and tracheal suction should be performed only in newborns who are not vigorous, and the decision to intubate and suction does not depend on the consistency of the meconium-stained fluid, as was once recommended. Because meconium is sterile, antibiotics are not needed. Steroids have not been necessary in the treatment of meconium-stained newborns. RDS is a condition that occurs as a result of low levels of pulmonary surfactant in the alveoli. RDS occurs in premature newborns, whereas meconium staining occurs typically in older, often post-term, newborns

40
Q
  1. A 38-year-old primiparous patient with placenta previa and active vaginal bleeding arrives in the operating room with a systolic blood pressure (SBP) of 85 mm Hg. A cesarean section is planned. The patient is lightheaded and scared. Which of the following anesthetic induction plans would be most appropriate for this patient?

A. Spinal anesthetic with 12 to 15 mg bupivacaine
B. General anesthetic induction with 2 to 2.8 mg/kg propofol and paralysis with 1 to 1.5 mg/kg succinylcholine
C. General anesthesia induction with 0.75 to 1 mg/kg ketamine and paralysis with 1 to 1.5 mg/kg succinylcholine
D. Replace lost blood volume first, then use any anesthetic the patient wishes

A

(C)Placenta previa occurs when the placenta implants on the lower uterine segment so that all (total) or part of the placenta (partial) covers the internal cervical os. A marginal placenta previa occurs when the placenta lies close to but does not cover the internal cervical os. Placenta previa occurs in about 0.5% of all pregnancies and has a maternal mortality of less than 1% but a fetal mortality approaching 20% (primarily because of prematurity and intrauterine asphyxia). Patients typically present with painless vaginal bleeding that stops spontaneously (first bleed). Delivery is cesarean and is often made a few weeks after the “first” bleed, when the baby’s lungs are more mature (e.g., after 37 weeks EGA). A later bleed can be uncontrolled and may be accompanied by significant hypovolemia and hypotension. Regional anesthesia is contraindicated in severely hypovolemic patients. Blood should be started when available, but replacing blood loss first to correct intravascular volume before induction of anesthesia may not be practical because bleeding may be faster than replacement is possible (i.e., may be >1unit/min). A rapid-sequence general anesthetic (assuming acceptable airway) is preferred. Ketamine (0.75-1mg/kg) as well as etomidate (0.3mg/kg) supports the cardiovascular system better than propofol. In rare but severe cases of hypovolemic shock, all IV anesthetics may cause the blood pressure to fall further, and succinylcholine alone may be all that is required. In these severe cases, maternal recall should be considered secondary to maternal safety. In cases in which a difficult intubation is likely and the patient is hypovolemic, an infiltration local anesthetic may be best

41
Q
  1. Which of the following lung volumes or capacities change the LEAST during pregnancy?

A. Tidal volume (Vt)
B. Functional residual capacity (FRC)
C. Expiratory reserve volume (ERV)
D. Vital capacity (VC)

A

(D)At term pregnancy, Vt increases about 40% to 45%, and the inspiratory reserve volume (IRV) increases about 5%. A decrease occurs in both the ERV (20%-25%) and the residual volume (RV; 15%-20%). A capacity is defined as two or more lung volumes. Functional residual capacity (FRC = ERV + RV) is decreased about 15% to 20% and is partly responsible for the rapid fall in maternal oxygenation that occurs with apnea during the induction of general anesthesia. Total lung capacity (TLC = Vt + IRV + ERV + RV) decreases about 5%, whereas vital capacity (VC = Vt + IRV + ERV) remains unchanged

42
Q
  1. General anesthesia is induced in a 35-year-old patient for elective cesarean section. No part of the glottic apparatus is visible after two unsuccessful attempts to intubate, but mask ventilation is adequate. The most appropriate step at this point would be to

A. Wake up the patient
B. Attempt a blind nasal intubation
C. Continue mask ventilation and cricoid pressure
D. Use a laryngeal mask airway

A

(A)Evaluation of the airway should be performed before the induction of any general anesthetic. In cases in which an unrecognized difficult airway exists (unable to perform endotracheal intubation in a reasonable period of time), the patient should be awakened if the procedure is purely elective and if the fetus has minimal or no fetal distress (as in this elective case). A regional anesthetic or awake intubation then can be safely performed. In cases of fetal or maternal distress, other options for securing the airway may be necessary

43
Q
  1. Which patients describe their labor pain as being the MOST intense?

A. Primipara patients attending prepared childbirth classes
B. Primipara patients not attending prepared childbirth classes
C. Multipara patients attending prepared childbirth classes
D. Multipara patients not attending prepared childbirth classes

A

(B)Labor pain is some of the most intense pain that people can experience. Although there is considerable variability in the amount of intensity of pain during labor, in general primiparous patients have more pain than multiparous patients. Primiparous women who have attended prepared childbirth classes have somewhat less pain than women who have not attended prepared childbirth classes. For women who have experienced labor and delivery, attending prepared childbirth classes does not seem to affect the amount of pain that they experience. Labor pain appears to exceed chronic low back pain, nonterminal cancer pain, phantom limb pain, postherpetic neuralgia, or the pain from a fracture. Patients with causalgia or patients experiencing an amputation of a digit have more pain than the parturient. There appears also to be an association of the increasing amount of pain with increasing cervical dilation.

44
Q
  1. Cigarette smoking is associated with an increase of each of the following EXCEPT

A. Spontaneous fetal loss
B. Placental abruption
C. Preeclampsia
D. Sudden infant death syndrome (SIDS)

A

(C)The prevalence of women smoking during pregnancy was about 16% in the United States in 2010. Cigarettes contain nicotine, carbon monoxide, cyanides, and over 4000 other chemicals. Smoking affects small airway function and mucus secretion, and it impairs ciliary transport in the lungs; smoking also increases atherosclerosis and heart disease. Nicotine increases sympathetic tone (e.g., maternal heart rate, blood pressure, cardiac work). Carbon monoxide can occupy 3% to 15% (or more) of the oxygen-carrying capacity of blood and thus decreases oxygen delivery to tissues. Smoking cessation, even for a short time, may be safer for the fetus. Nonpharmacologic methods are preferred over nicotine patches or nicotine gum to help women stop smoking during pregnancy because studies of nicotine replacement are lacking. Cigarette smoking is associated with an increase in ectopic pregnancies, placental abruption, spontaneous fetal loss, and low-birth-weight babies (with an increased neonatal and infant mortality), as well as SIDS and lower intelligence. Interestingly, the incidence of preeclampsia is 30% to 40% lower among women who are smokers compared with women who are nonsmokers. The reason for this is unclear but may be related to stimulation of nitric oxide release or an inhibition of thromboxane A2 synthesis. Because smoking irritates the airway, if general anesthesia is needed during pregnancy, desflurane is best avoided, whereas volatile anesthetics, such as sevoflurane, which does not irritate the airway, are preferred

45
Q
  1. All of the following are TRUE regarding the use of nitrous oxide for labor analgesia EXCEPT

A. Significant anxiolysis occurs
B. Do not need to have an IV line in place
C. Needs to be administered by anesthesia personnel
D. Only the patient can hold the mask or mouthpiece

A

(C)In 2012, the U.S. Food and Drug Administration (FDA) approved the use of simple devices for the administration of nitrous oxide as “minimal sedation” for labor analgesia. Inhaled nitrous oxide is used by <1% of patients for labor analgesia in the United States, but its use is more common in other countries such as Canada (43%) and the United Kingdom (62%). These noninvasive devices have demand valves where the patient must initiate the breathing. When inhalation is completed, there is no free flow of anesthetic into the room. The devices also have a scavenging system to decrease exposure to low levels of nitrous oxide. The gas inhaled is typically 50% nitrous oxide and 50% oxygen. The patient must hold the mask or mouthpiece in place without the assistance of a nurse, family member, or labor support person. Typically it takes about 30 to 60 seconds for analgesia to develop, so the patient should initiate inhalation at the start of the uterine contraction and continue breathing the gas until the end of the contraction. This gives the women in labor another analgesic option. The patients have the freedom to move about and do not require the placement of an IV or urinary catheter. It may prove useful for women with disorders of coagulation, chronic pain or anxiety, and a poor response to opioid medications. The quality of analgesia with inhaled nitrous oxide is variable, but less so than with epidural analgesia. Side effects of nitrous oxide for labor analgesia include nausea and vomiting (13%-33%), dizziness (4%), drowsiness (4%), and paresthesias (which are also related to maternal hyperventilation). The incidence of hypoxia appears to be low. Keep in mind that using narcotics or sedatives with nitrous oxide may depress respirations and increase the amount of nausea. Apgar scores of newborn infants whose mothers have used inhaled nitrous oxide appear to be similar to those of infants whose mothers used other forms or no forms of analgesia. Anesthesia personnel are not involved in their use

46
Q
  1. When performing a rapid-sequence induction (RSI) for an emergency cesarean delivery, which of the following muscle relaxants is LEAST desirable to use after the IV general anesthetic is administered?

A. Atracurium
B. Rocuronium
C. Succinylcholine
D. Vecuronium

A

(A)When performing an RSI for a cesarean section, the patient is preoxygenated, and after you ascertain that intubation should be easy, you inject your IV general anesthetic (e.g., propofol 2.5mg/kg, ketamine 1-1.5mg/kg, or etomidate 0.3mg/kg) followed by a paralytic drug to facilitate oral endotracheal intubation. Thiopental (4-5mg/kg) is another IV general anesthetic that can be used if it is available (not available in the United States). Paralysis to optimize tracheal intubation conditions can be obtained with either succinylcholine (1-1.5mg/kg in 30-45 seconds) or rocuronium (1.2mg/kg in <60 seconds). Vecuronium (0.1mg/kg) can also be used but is less desirable because it has a slower onset of action of 144 seconds. Atracurium is the least desirable of the listed drugs due to the significant histamine release and resultant hypotension and slow onset of action. Although cisatracurium has little histamine release, it has a very slow onset of action and is not recommended for an RSI induction if succinylcholine, rocuronium, or vecuronium is available

47
Q
  1. True statements regarding inclusion of intrathecal morphine, fentanyl, or sufentanil in obstetric anesthesia practice include each of the following EXCEPT

A. The chief site of action is the substantia gelatinosa of the dorsal horn of the spinal column
B. There is no motor and no sympathetic blockade
C. Pain relief is adequate for the second stage of labor
D. Lipophilic narcotics are associated with less respiratory depression than nonlipophilic narcotics

A

(C)Intrathecal opiates (e.g., morphine, fentanyl, sufentanil) are very effective in relieving the visceral pain during the first stage of labor. Intrathecal opiates administered alone (except for meperidine, which has local anesthetic properties) do not provide adequate pain relief for second-stage somatic pain

48
Q
  1. The MOST common side effect of intraspinal narcotics in the obstetric population is

A. Pruritus
B. Nausea and vomiting
C. Respiratory depression
D. Urinary retention

A

(A)The most common side effect of intraspinal narcotics is pruritus. The next most common side effects are nausea and vomiting, followed by urinary retention and drowsiness. Respiratory depression and headache may occur but are relatively infrequent

49
Q
  1. A 110-kg (242-lb), gravida 1, para 0 woman has a blood pressure of 180/95 during an office visit at the 16th week of gestation and 175/90 1 week later. She has some ankle but no facial edema, and no protein detected in her urine. Her serum creatinine is 1.2. These findings would be classified as

A. Preeclampsia
B. Chronic hypertension
C. Chronic hypertension with superimposed preeclampsia
D. Gestational hypertension

A

(C)Hypertension during pregnancy is divided into four groups: preeclampsia-eclampsia, chronic hypertension (of any cause), chronic hypertension with superimposed preeclampsia, and gestational hypertension. Preeclampsia-eclampsia is a hypertensive disorder of pregnancy with associated proteinuria (≥300mg protein per 24-hour urine collection). As of recently (November 2013), the presence of proteinuria is no longer needed for the designation of preeclampsia-eclampsia. The reason for the change is that some patients develop proteinuria late and have their diagnosis and needed treatment delayed. Current definition of preeclampsia-eclampsia is the new onset of hypertension along with proteinuria or, in the absence of proteinuria, new onset of hypertension associated with thrombocytopenia (platelet count <100,000/mL), impaired liver function, new onset of renal insufficiency (serum creatinine >1.1mg/dL or doubling of serum creatinine in the absence of any other renal disease), pulmonary edema, or new-onset cerebral or visual disturbances. Gestational hypertension, which is isolated new-onset hypertension (after 20 weeks) that resolves by 12 weeks postpartum, is a retrospective diagnosis. Preeclampsia-eclampsia rarely occurs before the 20th week of pregnancy, except in patients with gestational trophoblastic neoplasms (e.g., molar pregnancy). The incidence of preeclampsia is significantly higher in parturients with a hydatidiform mole, multiple gestations, obesity, polyhydramnios, or diabetes and occurs more commonly with the first pregnancy. Mothers with preeclampsia during their first pregnancy have a 33% chance of having preeclampsia in subsequent pregnancies. Preeclampsia can progress to eclampsia (preeclampsia accompanied by a seizure not related to other conditions). Eighty percent of the seizures occur before or during delivery; 85% of the remaining 20% will have the seizure within the first 24 hours after delivery. Approximately 5% of untreated parturients with preeclampsia will develop eclampsia. HELLP syndrome (Hemolysis, Elevated Liver enzymes, and Low Platelet count) is a variant of preeclampsia. Chronic hypertension is persistent hypertension before, during, and after pregnancy (e.g., >6 weeks postpartum). Chronic hypertension with superimposed preeclampsia occurs when a patient with chronic hypertension develops preeclampsia. Also see Question 645

50
Q
  1. An epidural is placed into a 32-year-old parturient in active labor receiving magnesium therapy for preeclampsia. Five minutes after administration of the test dose, the loading dose of bupivacaine and fentanyl is administered. The patient becomes panic-stricken, wrestles briefly with the reassuring nurses, gasps for air, seizes, and develops cardiovascular collapse. During resuscitation, blood is oozing from the IV sites and a pink froth is noted in the endotracheal tube. The MOST likely diagnosis is

A. Amniotic fluid embolism
B. High spinal
C. Intravascular bupivacaine injection
D. Eclampsia

A

(A)AFE is a rare condition (5 per 100,000 live births). It presents in a variety of ways but often in a dramatic way, with acute hypoxemia, cardiovascular collapse, DIC, and, in about 50% of cases, a seizure. Patients with a high spinal or epidural may complain of dyspnea, but they also have marked weakness and would certainly not be able to wrestle or struggle with their health care providers. Patients experiencing an intravascular injection of local anesthetic present with CNS signs of toxicity (light-headedness, visual or auditory disturbances, muscular twitching, convulsion, coma) or, at higher levels, cardiovascular collapse. Magnesium overdosage is also associated with muscle weakness. The typical eclamptic seizure is tonic–clonic. Patients with eclampsia do not complain of dyspnea, although an associated aspiration may produce similar symptoms. See Question 658

51
Q
  1. Which of the following narcotics has the LONGEST duration of action when added during a cesarean section under epidural anesthesia?

A. 50 to 100 μg fentanyl
B. 10 to 20 μg sufentanil
C. 3 to 4 mg morphine
D. 50 to 75 mg meperidine

A

(C)Epidural fentanyl (50-100μg) and epidural sufentanil (10-20μg) each have a duration of action for about 2 to 4 hours. Epidural meperidine (50-75mg) has an intermediate duration of action of 4 to 12 hours, whereas epidural morphine (3-4mg) has the longest duration of action, of 12 to 24 hours

52
Q
  1. Which of the following is NOT increased during pregnancy?

A. Renal plasma flow
B. Creatinine clearance
C. Blood urea nitrogen (BUN)
D. Glucose excretion

A

(C)The renal system undergoes dramatic anatomic (increase in kidney size as well as dilation of the ureters) and functional changes in pregnancy. Renal plasma flow increases about 75% to 85%, and glomerular filtration rate (GFR) increases about 50% and is reflected by an increase in clearance of urea, creatinine, and uric acid. Because of the increased clearance, we see a decrease in BUN to 8 to 9mg/dL, serum creatinine to 0.5 to 0.6mg/dL, and serum urate to 2.0 to 3.0mg/dL. Glucosuria is common and is attributed to both the increase in GFR and a reduced renal tubular resorption of glucose

53
Q
  1. Which inhalation anesthetic does NOT produce uterine relaxation?

A. Isoflurane
B. Sevoflurane
C. Nitrous oxide
D. All produce uterine relaxation

A

(C)All volatile halogenated anesthetic agents (e.g., halothane, enflurane, isoflurane, desflurane, sevoflurane) cause a dose-related relaxation of uterine smooth muscle. With anesthetic concentrations of 0.2 MAC, the decrease in uterine activity is slight, and these agents have been used for inhalation analgesia during labor. At 0.5 MAC, uterine relaxation is more significant, but the uterine response to oxytocin remains intact. Nitrous oxide does not affect uterine activity

54
Q
  1. Passive diffusion of substances across the placenta is enhanced by all of the following EXCEPT

A. Low molecular weight of the substance
B. High water solubility of the substance
C. Low degree of ionization of the substance
D. Large concentration gradient of the drug

A

(B)Passive diffusion is the primary means for the placental transfer of drugs. Factors that promote diffusion of drugs across placental membranes include decreased maternal protein binding (although some believe that this is not very important because of rapid diffusion of drugs from protein), low molecular weight (<500Da), high lipid solubility (low water solubility), a low degree of ionization, and a large concentration gradient across the membranes. Highly ionized drugs, such as neuromuscular drugs, do not pass the placenta in significant amounts

55
Q
  1. Cesarean delivery is associated with a blood loss of about

A. 250 mL
B. 500 mL
C. 750 mL
D. 1000 mL

A

(D)The average blood loss associated with a vaginal delivery is about 600mL and after a cesarean delivery is about 1000mL (Chestnut: Chestnut’s Obstetric Anesthesia, ed 5, pp 24–25).

55
Q
  1. Which of the following statements is CORRECT in describing differences between fetal and maternal blood during labor?

A. Fetal blood has a lower hemoglobin concentration than does maternal blood
B. Fetal placental blood flow is twice maternal placental blood flow
C. Fetal hemoglobin has a greater affinity for O2 than does maternal hemoglobin
D. The fetal oxyhemoglobin dissociation curve is shifted to the right of the maternal oxyhemoglobin dissociation curve

A

(C)The fetus has several compensatory mechanisms for dealing with low O2 pressures (umbilical vein Po2 approximately equal to 30mm Hg when the mother is breathing room air) to which it is exposed. These include a higher hemoglobin concentration (15-20g/dL) and the presence of fetal hemoglobin, which has a greater affinity for oxygen (the fetal oxyhemoglobin dissociation curve is shifted to the left of the maternal oxyhemoglobin dissociation curve). At term, maternal blood flow through the placenta (700mL/min) is about double the fetal blood flow through the placenta (300-360mL/min). Fetal blood has a lower pH than maternal blood, which may be related to the higher Paco2 levels seen in fetal blood

56
Q
  1. In general, morbidly obese patients have a higher incidence of all of the following EXCEPT

A. Cesarean deliveries
B. Postdural puncture headaches (PDPHs)
C. Preeclampsia
D. Thromboembolic diseases

A

(B)A pregnant woman is considered overweight when her body mass index (BMI) is 25.0 to 29.9kg/m2 and obese when the BMI is greater than 30kg/m2. The World Health Organization (WHO) defines three grades of obesity: Class I = BMI 30 to 34.5kg/m2, Class II as BMI 35 to 39.9kg/m2, and Class III as BMI ≧40kg/m2. The obese patient is at increased risk for several comorbid diseases, including obstructive sleep apnea (OSA), diabetes, hypertension, gallbladder disease, chronic back pain, and coronary artery disease. During pregnancy, the obese patient has an increased chance of gestational diabetes, hypertensive disorders of pregnancy, preterm delivery, deep vein thrombosis, pulmonary thromboembolism, respiratory depression, wound infections, postpartum hemorrhage, cesarean deliveries, and death (reviews note that about 50% or more of pregnant women who die are overweight or obese). The increased incidence of cesarean deliveries may relate to an increase in abnormal presentations, fetal macrosomia, meconium staining, late decelerations in the FHR, and dysfunctional labor. Anesthetic challenges include increased risk of aspiration, difficulty finding adequate venous access, difficulty with mask ventilation, difficulty with endotracheal intubation, difficulty in performing regional anesthesia, operative positioning, and prolonged surgery. Interestingly, obese and morbidly obese patients appear to have a lower incidence of PDPHs. Etiology for the lower incidence of PDPHs is unclear

57
Q
  1. A term infant with good muscle tone and a strong cry has an oxygen saturation of 83%, breathing room air 5 minutes after delivery. The MOST appropriate action at this point would be

A. Supplemental increased oxygen concentration with a blender up to 50% by a face mask
B. Spontaneous breathing with 100% oxygen by face mask
C. Positive-pressure ventilation with 100% oxygen
D. Observation

A

(D)Normal healthy term newborns breathing room air take a while for the oxygen saturations to rise to normal 90% to 95% levels. In caring for the newborn who is not breathing, bag and mask ventilation with room air is now recommended, with targeted preductal oxygen saturation (right hand or wrist) increases of about 5% for each minute of the first 5 minutes of life, starting at 1 minute oxygen saturation of 60% to 65% (at 2 minutes 65%-70%, at 3 minutes 70%-75%, at 4 minutes 75%-80%, at 5 minutes 80%-85%). After 5 minutes, oxygen saturation more slowly increases to 85% to 95% by 10 minutes of life. If higher concentrations of oxygen are needed to reach the targeted oxygen saturations (especially in preterm newborns <32 weeks), a blender for oxygen and air can be used. For this newborn, an oxygen saturation of 83% at 5 minutes is appropriate, and observation only is needed

58
Q
  1. Which condition BEST describes the third-trimester maternal condition with the following signs and symptoms: new-onset vaginal bleeding that stops, no pain, no fetal distress?

A. Placental abruption
B. Placenta previa
C. Uterine rupture
D. Vasa previa

A

(B)Second- and third-trimester obstetric hemorrhage is not uncommon in obstetrics. Placenta previa (where the placenta is partially or completely covering the cervical os) is suspected when painless vaginal bleeding occurs during the second or third trimester. This first episode of bleeding is usually not associated with maternal shock or fetal distress and is expectantly managed if the bleeding stops and the fetus is immature. However, with a second or third episode of vaginal bleeding, the bleeding may continue and delivery may be indicated. Placental abruption (separation of the placenta from the uterine wall after 20 weeks’ EGA and before delivery) is more typically associated with abdominal pain and can be associated with fetal distress. Bleeding with placenta abruption may be revealed or concealed behind the placenta. Uterine rupture usually presents with severe abdominal pain and fetal distress. Vasa previa refers to a condition where the fetal blood vessels are unsupported by the umbilical cord or placental tissue and cross over the cervical os. When the fetal membranes rupture, a tear in a fetal blood vessel may develop, leading to fetal exsanguination

59
Q
  1. During the second stage of labor, complete pain relief can be obtained with

A. Paracervical block
B. Neuraxial block with fentanyl and morphine
C. Pudendal nerve block
D. Lumbar epidural block with bupivacaine and no narcotic

A

(D)The first stage of labor starts with the onset of labor and ends with complete cervical dilation (10cm). It is visceral pain, associated with uterine contractions and dilation of the cervix, and is transmitted via the autonomic nervous system through the sympathetic fibers that pass through the paracervical region and enter the CNS at T10-L1 segments. The second stage of labor includes these pathways and adds the somatic fibers of the birth canal that are transmitted via the pudendal nerve entering the CNS at S2-S4. Neuraxial block (spinal and/or epidural) with only narcotics can be useful for first-stage pain; however, the somatic pain is not well treated with narcotics alone. A local anesthetic–induced lumbar epidural block with or without narcotics can produce complete anesthesia during both first and second stage of labor pain. If a low spinal or saddle block is performed with local anesthetics (covering only sacral areas), the uterine contraction pain still will be felt. Paracervical blocks block only the first-stage pain. Pudendal blocks block the somatic component during the second stage but not the visceral pain of uterine contractions

60
Q
  1. Anesthetic considerations for open fetal surgery include all of the following EXCEPT

A. Uterine relaxation is essential
B. Maternal hypotension (mean blood pressure < 65 mm Hg) can be treated with phenylephrine or ephedrine
C. Vecuronium at the ED95 dose of 0.04 mg/kg should be administered IM or IV by the obstetrician or surgeon if fetal muscle relaxation is needed
D. Normal fetal oxygen saturation is 50% to 70%

A

(C)Anesthetic considerations for open fetal surgery include administering anesthesia for the mother and the child, giving excellent uterine relaxation, maintaining an adequate maternal blood pressure, providing muscle relaxation to the fetus if needed, and preventing postoperative premature labor. Uterine relaxation is needed to prevent uterine contractions, with possible separation of the placenta from the uterine wall. High-dose volatile anesthetics (e.g., 2 or 3 MAC) can provide excellent maternal anesthesia, as well as uterine relaxation and anesthesia for the fetus. If additional anesthesia is needed or when a lower concentration of volatile anesthetic is administered, maternally administered IV narcotics can be used (e.g., remifentanil infusions at a dose of 0.1 μg/kg/min) or a fetal IM injection of fentanyl (up to 10-20 μg/kg) can be performed. If one chooses to use a lower dose of volatile anesthetics, nitroglycerin infusion may be needed to keep the uterus from contracting. Maternal hypotension (mean blood pressure <65) is not uncommon and is treated with more left uterine tilt, fluids, and, if needed, phenylephrine or ephedrine. Monitoring the fetal oxygen saturation reveals normal values of 50% to 70%; values less than 50% signal impaired placental perfusion (e.g., maternal hypotension, cord compression). If the obstetrician needs the fetus to be paralyzed, then a neuromuscular blocking drug can be given directly into the fetus because placental transfer of muscle relaxants is poor. The dose of muscle relaxant, however, must be larger when the fetus is in utero compared with the fetus after delivery (newborn of the same weight) because the blood volume of the fetus in utero includes the fetal blood volume as well as the placental blood volume. Typically the IM dose is about four to six times the effective dose in 95% of subjects (ED95) or for vecuronium is 0.2 to 0.3mg/kg. Magnesium sulfate may be started to decrease the chance of premature labor at the end of the surgery as the volatile anesthetic concentration is decreased or the nitroglycerin infusion is discontinued. One should recall that the magnesium sulfate potentiates neuromuscular blocking drugs significantly. In many cases of fetal surgery, the mother may also receive an epidural for postoperative analgesia

61
Q
  1. 15-Methyl PGF2α is administered directly into the myometrium to treat uterine atony in a 28-year-old mother. Possible effects from treatment with this drug include

A. Nausea and vomiting
B. Bronchospasm
C. Hypoxia
D. All of the above

A

(D)15-Methyl PGF2α (carboprost, Hemabate) can be used for the treatment of refractory uterine atony (after oxytocin is administered and uterine massage has been performed). It works by increasing contractions of smooth muscle. The dose is 0.25mg injected IM or directly into the uterine wall, repeated as needed every 15 to 90 minutes with a maximum total dose of 2mg (8 doses). It has several important side effects, such as bronchospasm, ventilation-to-perfusion mismatch with an increase in intrapulmonary shunting, hypoxemia, and increased blood pressure. It is contraindicated in patients with asthma and relatively contraindicated in patients with pregnancy-induced hypertension. Other side effects include GI spasms (e.g., nausea, vomiting, and diarrhea)

62
Q
  1. Which of the following statements regarding MgSO4 therapy for preeclampsia is TRUE?

A. The therapeutic range for serum magnesium is 10 to 15 mEq/L
B. High serum magnesium levels can be estimated by changes in deep tendon patellar reflexes in a patient with an epidural anesthetic loaded for a cesarean section
C. Excessive serum magnesium levels cause widening of the QRS complex
D. As soon as delivery occurs, the chance for eclampsia no longer exists and the magnesium should be reversed so that postpartum bleeding is less likely to occur

A

(C)International consensus states that magnesium sulfate (MgSO4) is the anticonvulsant of choice in the preeclamptic patient. In addition to its anticonvulsant effect, MgSO4 has many other actions on skeletal and cardiac muscles. MgSO4 is usually started as an IV bolus of 6g over 20 minutes followed by an infusion of 2g/hr (provided that kidney function is normal). Clinical monitoring for toxicity is performed looking at deep tendon reflexes, and blood levels are often performed and reported in either mEq/L or mg/dL (1mEq/L = 1.22mg/dL). The therapeutic range for serum MgSO4 is 4 to 8mEq/L (4.8-9.6mg/dL). In an unanesthetized patient, a loss of deep tendon reflexes occurs at 10mEq/L (12mg/dL), respiratory arrest occurs at 15mEq/L (18mg/dL), and cardiac arrest occurs at 25mEq/L (30mg/dL). As long as deep tendon reflexes are present, significant toxicity is unlikely. In a patient with an epidural or spinal anesthetic loaded for a cesarean section, the patellar reflex is often depressed by the local anesthetic; estimation of deep tendon reflexes should be done with the biceps tendon (unless a total spinal develops). ECG changes, including PR interval prolongation and QRS complex widening, occur at serum levels of 5 to 10mEq/L (6-12mg/dL), sinoatrial and atrioventricular block at 15mEq/L (18mg/dL), and cardiac arrest at levels greater than 25mEq/L (30mg/dL). The treatment for magnesium toxicity is calcium. The dose of 1g of calcium gluconate (10mL of a 10% solution) can be administered slowly over at least 2 minutes to treat high magnesium levels. Rapid administration may take away the anticonvulsant effects, so careful slow titration is recommended. About 60% of eclamptic seizures occur before delivery. Most postpartum seizures develop in the first 24 hours after delivery, but eclamptic seizures may occur as late as 22 days after delivery

63
Q
  1. While moving a parturient from the birthing room to the operating room for an emergency cesarean section for a prolapsed umbilical cord, the patient develops cough, wheezing, and stridor and becomes cyanotic. The trachea is intubated, and food is noted in the pharynx. Appropriate treatment in this patient should consist of

A. Intravenous lidocaine to suppress the cough
B. Glucocorticoids
C. 100% oxygen and positive end-expiratory pressure (PEEP)
D. Saline lavage

A

(C)Three different aspiration syndromes have been described in the general population: aspiration of particulate matter causing mainly mechanical airway obstruction, aspiration of acid fluid causing aspiration pneumonitis (Mendelson syndrome), and aspiration of gram-positive, gram-negative, and anaerobic bacteria (e.g., bowel obstruction) causing aspiration pneumonia. Symptoms of aspiration pneumonitis include coughing, tachypnea, tachycardia, bronchospasm, and hypoxemia. Treatment is supportive and includes the Heimlich maneuver if a large foreign body is lodged in the trachea (which is unlikely in the fasting laboring patient), endotracheal intubation, suctioning the airway to remove particulate material, administration of increased concentrations of oxygen, and application of PEEP to achieve oxygenation goals as needed. Coughing is due to the airway irritation and is most effectively decreased with muscle paralysis. Intravenous lidocaine is not effective. Use of saline or bicarbonate lavage does not decrease lung damage and can worsen hypoxemia. Glucocorticoids or other anti-inflammatory drugs have not been effective in limiting the inflammation and may increase the risk of secondary bacterial infection

64
Q
  1. Aortocaval compression starts to become significant in a normal pregnancy at how many weeks EGA?

A. 10 weeks
B. 15 weeks
C. 20 weeks
D. 25 weeks

A

(C)Aortocaval compression typically is not a problem until about 18 to 20 weeks’ gestation, when the uterus is large enough to compress the aorta and vena cava when the patient assumes the supine position. If the uterus is larger than normal (e.g., multiple gestations or polyhydramnios), then aortocaval compression may appear earlier. See also Question 714

65
Q
  1. Which agent is the MOST useful for raising the gastric pH just before induction of general anesthesia for emergency cesarean section?

A. Ranitidine
B. Sodium citrate
C. Metoclopramide
D. Magnesium hydroxide and aluminum hydroxide

A

(B)Cimetidine and ranitidine are H2-receptor antagonists that will increase gastric pH but take at least 30 minutes to work. Metoclopramide is not an antacid but may be useful by increasing the lower esophageal sphincter tone. Only liquid antacids raise gastric pH quickly. Sodium citrate, a clear nonparticulate antacid (0.3M sodium citrate), is preferred over particulate antacids (aluminum hydroxide, magnesium trisilicate, magnesium hydroxide) because clear nonparticulate antacids cause less pulmonary damage if aspirated. Sodium citrate 30mL neutralizes 255mL of HCl with a pH of 1.0. Neutralization of gastric acid occurs rapidly (i.e., <5 minutes) and will last up to an hour

66
Q
  1. Causes of fetal bradycardia include all of the following EXCEPT

A. Maternal smoking of cigarettes
B. Neostigmine and glycopyrrolate reversal of neuromuscular blockers
C. Acidosis
D. Umbilical cord compression

A

(A)Causes of fetal bradycardia (FHR <110beats/min) include hypotension, excessive uterine activity, hypoxemia, acidosis, complete heart block, and some drugs. Atropine readily crosses the placenta but at low doses does not seem to cause fetal tachycardia; at high doses, it may produce tachycardia. The combination of neostigmine, which crosses the placenta slightly, and glycopyrrolate, which does not cross the placenta well, has been associated with fetal bradycardia, which is why neostigmine with atropine is preferred when reversing neuromuscular blockers if a fetus is present. Fetal bradycardias are often associated with early decelerations (head compression with vagal stimulation), late decelerations (fetal hypoxemia with vagal stimulation or myocardial failure), and variable decelerations (umbilical cord compressions with vagal stimulation). Causes of fetal tachycardia (FHR >160beats/min) include infection, fever, maternal cigarette smoking, fetal paroxysmal supraventricular tachycardia, and some drugs (ritodrine, terbutaline, atropine)

67
Q
  1. Most cases of cerebral palsy (CP) are due to conditions during

A. Antepartum
B. Labor
C. Delivery
D. The first 30 days of life

A

(A)CP is a nonprogressive disorder of the CNS arising from lesions in the brain that occurred during development (in utero 75%, at birth 10%, soon after birth 15%). CP is associated with impairment of motor function. Mental retardation may or may not be present and is not an essential diagnostic criterion. The cause is unknown and most likely multifactorial. Associated conditions include maternal mental retardation (now called intellectual disability), birth weight of less than 2000g and fetal malformations, breech presentation (but not breech vaginal delivery), severe proteinuria during the second half of pregnancy, third-trimester bleeding, and gestational age less than 32 weeks, but many other factors may play a role. It occurs in about 2 per 1000 live births. At one time, FHR monitoring was thought to be able to prevent CP, but this has not happened. In fact, among patients with new-onset late deceleration patterns, the false-positive rate is 99% if used to predict the development of CP. This is not to say that intrapartum asphyxial insults do not cause damage; they might, and they probably account for some cases of CP. There is also a very weak association of low Apgar scores and CP; in fact, most children who develop CP had a 5-minute Apgar score that was normal

68
Q
  1. All of the following statements regarding pregnant diabetic patients are true EXCEPT

A. Gestational diabetes mellitus (DM) occurs in about 7% of all pregnancies in the United States
B. Insulin readily crosses the placenta and causes larger babies
C. Cesarean section is more common in diabetic pregnancies
D. Diabetic ketoacidosis (DKA) occurs in 1% to 2% of Type 1 DM pregnancies

A

(B)Seven to eight percent of the adult population in the United States has DM. Type 1 DM (about 10% of diabetics) is a T-cell–mediated autoimmune disease where the patients have an absolute deficiency in insulin secretion. Type 2 DM (about 90% of diabetics) occurs primarily in obese individuals and is associated with an inadequate release of insulin or insulin resistance in target tissues. Type 1 DM occurs in 1 of every 700 to 1000 gestations. Gestational diabetes, which occurs only during pregnancy, is currently seen in about 7% of all pregnancies in the United States. Although substantial advances in the obstetric and anesthetic management of diabetic parturients have been made, maternal and fetal mortality is still higher in these patients than in parturients without diabetes. DKA occurs primarily in Type 1 DM patients and has decreased from 9% to currently around 1% to 2% of Type 1 DM pregnancies. One important goal of insulin therapy in diabetic patients is to avoid both hyperglycemia and hypoglycemia. In general, insulin requirements in Type 1 DM patients initially decrease during early pregnancy to their lowest requirement by around 16 to 18 weeks (10%-20% reduction in dose), then increase above prepregnant values around 26 weeks to reach values that are highest at term (50% above prepregnant dose). The dose requirements then rapidly decrease at the time of delivery. Insulin does not readily cross the placenta and therefore does not have any direct effects on glucose metabolism in the fetus. Glucose, however, readily crosses the placenta. Preeclampsia and large-for-gestational-age fetuses occur more frequently in parturient women with diabetes. Because of fetal macrosomia, cesarean section is more common in diabetic than nondiabetic patients

69
Q
  1. In addition to the postural component of a postdural puncture headache (PDPH), signs and symptoms may include any of the following EXCEPT

A. Double vision
B. Hearing changes
C. Neck stiffness
D. Fever

A

(D)PDPHs are positional headaches (exacerbated by sitting or standing and relieved with recumbency) that usually present within 48 hours of a dural puncture (but could take up to a week to present) and typically resolve in 2 to 14 days. They are bilateral and typically located in the frontal or occipital regions. In one prospective series of nonobstetric patients with PDPH, symptoms included nausea (60%), vomiting (24%), neck stiffness (43%), ocular changes (photophobia, diplopia, difficulty in accommodation) (13%), and auditory changes (hearing loss, hyperacusis, tinnitus) (12%). Although postpartum seizures have been associated with PDPH, other etiologies are more likely. Seizures, lethargy, fever, nuchal rigidity, focal neurologic deficits (other than listed above), and a unilateral location suggest other headache etiologies

70
Q
  1. Early decelerations may occur in response to

A. Fetal head compression
B. Uteroplacental insufficiency
C. Maternal hypotension
D. Umbilical cord compression

A

(A)There are several periodic FHR patterns. Accelerations in FHR in response to fetal movement signify fetal well-being. Decelerations are a decrease in FHR of at least 15beats/min that last at least 15 seconds. Early decelerations are decreases in FHR that are usually less than 20beats/min and occur concomitantly with uterine contractions. Typically they are smooth and are mirror images of the uterine contractions. They are not associated with fetal compromise and are caused by head compression, which produces a vagal slowing of the FHR. Late decelerations are decreases in FHR that occur 10 to 30 seconds after the onset of a contraction and end 10 to 30 seconds after the end of a contraction. They are due to uteroplacental insufficiency and can result whenever uterine blood flow decreases. The delayed onset is due to the time required to sense a low oxygen tension. The decrease in FHR may be a vagal reflex (mild cases) or may be due to direct myocardial depression from hypoxia (severe cases). Typically, in severe cases, beat-to-beat variability is decreased or absent as well. Variable decelerations are abrupt decreases in FHR that vary in shape, depth, and duration from contraction to contraction. They are thought to be due to transient umbilical cord compression. A sinusoidal pattern is a regular smooth wavelike pattern with no short-term variability. It may be caused by severe fetal anemia or may result from the maternal administration of narcotics

71
Q
  1. Agents that are useful for decreasing the incidence of shivering during cesarean section under regional anesthesia or for treating shivering include all of the following EXCEPT

A. Administration of intrathecal local anesthetic with fentanyl and/or morphine
B. Intravenous magnesium sulfate
C. Administration of epidural local anesthetic solutions with epinephrine
D. Intravenous meperidine

A

(C)Shivering occurs in 15% to 20% of all normal vaginal deliveries. The frequency increases from 20% to 85% of patients receiving epidural or spinal anesthesia for cesarean deliveries. The postulated reason is that neuraxial anesthesia impairs centrally mediated peripheral vasoconstriction and shivering thresholds and allows greater environmental heat loss (core to peripheral heat redistribution). Intrathecal narcotics (e.g., especially fentanyl with morphine) and epidural narcotics (e.g., fentanyl, sufentanil, meperidine, butorphanol), when added to local anesthetics, decrease the incidence of maternal shivering. Intravenous meperidine (25mg), clonidine (75μg), ketanserin (10mg), magnesium sulfate (30mg/kg), or dexmedetomidine decrease the incidence of shivering. Warming the epidural anesthesia solution to body temperature has no effect on the incidence of shivering; however, adding epinephrine to the local anesthetic appears to increase the frequency of shivering

72
Q
  1. An umbilical arterial blood gas sample at the time of an emergency cesarean delivery shows a Po2 of 20 mm Hg, a Pco2 of 50 mm Hg, a bicarbonate value of 22 mEq/L, and a pH of 7.25. This shows

A. Severe hypoxemia
B. Respiratory acidosis
C. Metabolic acidosis
D. Normal values

A

(D)The values listed in the question are normal umbilical cord values. The chart is modified from values listed in the references

73
Q
  1. Which is the MOST frequent condition requiring blood transfusions during or after a cesarean delivery?

A. Multiple gestations
B. Placental abruption
C. Placenta previa
D. Postpartum hemorrhage

A

(D)For all obstetric-related admissions, the incidence of transfusion of blood is less than 1%. The most common reason for transfusion was postpartum hemorrhage. Estimates suggest that about one third of transfusions were not appropriate with current guidelines

74
Q
  1. All of the following are appropriate techniques or drug doses to be used in resuscitating a depressed term newborn EXCEPT

A. Begin ventilation with air rather than 100% oxygen
B. If the heart rate is less than 60 beats/min, start chest compressions (ratio of chest compressions to ventilations is 3:1)
C. After adequate ventilation and chest compressions, administer 0.1 mg/kg of epinephrine IV
D. After 10 minutes of no detectable heart rate, it may be reasonable to discontinue resuscitation efforts

A

(C)Resuscitation guidelines continue to evolve. About 10% of newborns require some resuscitation to assist breathing at birth, whereas about 1% will need extensive resuscitation. If the newborn appears to be term, has good muscle tone, and is breathing or crying, the baby can remain with the mother and can have initial care performed on mother’s chest or abdomen. Initial care consists of drying the newborn, providing warmth, and clearing secretions if needed. If the newborn appears depressed, next open the airway, clear out any remaining oral secretions, then stimulate the newborn by rubbing a towel to completely dry the newborn. If after a minute there is an obvious need for more extensive treatment (e.g., the heart rate is less than 100beats/min or the newborn is gasping or apneic), then positive-pressure ventilation is started and oximetry and ECG monitoring are suggested. In term newborns, resuscitation is started with air rather than with 100% oxygen. However, preterm newborns (<32 weeks’ gestation) may need air blended with oxygen to reach adequate oxygen saturations (e.g., at 10 minutes after delivery, the oxygen saturation should be 85%-95%). The oximetry probe should be placed preductally (i.e., on the right wrist) to assess oxygenation. If, after the first minute of life, the heart rate is less than 100beats/min, one should ensure adequate positive-pressure mask ventilation and consider endotracheal intubation (or the use of a laryngeal mask). The ventilation rate should be 40 to 60 breaths per minute. If the heart rate is now less than 60beats/min after at least 30 seconds of good positive-pressure ventilation, endotracheal intubation should be performed and 100% oxygen should be used. Chest compressions should then begin (with a chest compression-to-ventilation ratio of 3:1). At this point the newborn receives 30 breaths and 90 compressions/min (e.g., one and two and three and breath). If the newborn is known to have a cardiac etiology, then a higher compression-to-ventilation ratio should be considered (e.g., 15:2). If the heart rate is less than 60beats/min after chest compressions and positive-pressure ventilation have been started for at least 30 seconds, consider administering epinephrine. The correct dose is 0.01 to 0.03mg/kg IV or intraosseous (IO) and can be given every 3 to 5 minutes if needed. If the newborn is intubated and IV or IO access has not yet been achieved, consider administering a higher dose of epinephrine such as 0.05 to 0.1mg/kg down the endotracheal tube (the higher dose is used because blood levels are unpredictable after endotracheal instillation). In a newborn with blood loss, volume expansion is needed and can be achieved with normal saline, Ringer lactate, or type O Rh-negative blood. The initial dose is 10mL/kg. There is little evidence of any benefit with volume expansion in the absence of blood loss. If after 10 minutes there is no detectable heart rate, it may be appropriate to discontinue resuscitation (although many factors can contribute to continuing resuscitation beyond 10 minutes

75
Q
  1. After a vaginal delivery under epidural anesthesia, a healthy 8-lb baby is born. The 23-year-old now gravida 1, para 1 woman is noted to have a temperature of 38.2° C. A leukocyte count is obtained and is 15,000/mm3. The most appropriate course of action would be to

A. Get a blood culture
B. Start antibiotics
C. Administer a sedative
D. Observe

A

(D)Although epidural anesthesia often causes a fall in body temperature (due to the vasodilation and redistribution of body heat and loss to the environment), some women develop a rise in body temperature even though there is no evidence of infection. This rise in body temperature of greater than 38° C (100.4° F) usually occurs only when the epidural was used for at least 4 to 5 hours (frequency of 1%-36% of patients). The etiology of this rise in temperature in some women is unclear but includes three main factors (thermoregulatory, effect of systemic opioids, and inflammation). Epidural anesthesia may decrease sweating and the hyperventilation associated with labor, as well as shivering, which may increase body temperature. The use of IV systemic opioids may decrease the incidence of fever. Inflammation may play an important role because maternal temperatures are similar in women with or without epidural anesthesia when histologic examination of the placentas reveals the absence of placental inflammation. It may be that the temperature rise was merely an association with obstetric factors such as nulliparity with prolonged labor, more frequent cervical examinations, prolonged rupture of membranes, or early chorioamnionitis. The prepregnant blood leukocyte count of 6000/mm3 rises during pregnancy to 9000 to 11,000/mm3. During labor the leukocyte count increases to 13,000/mm3, and during the first postpartum day is on average 15,000/mm3

76
Q
  1. Compared with a healthy 25-year-old primigravida, which of the following conditions is NOT associated with a significantly higher incidence of hypertensive disorders of pregnancy?

A. Multiple gestations
B. Cigarette smoking (> 1 pack/day)
C. Obesity
D. Placental abruption

A

(B)Although the cause of hypertensive disorders of pregnancy is not known, many factors are associated with a higher frequency of hypertensive disorders of pregnancy. Factors include nulliparous woman, age (especially <20 years and >40 years), family history of hypertensive disorders of pregnancy or a previous history of hypertensive disorders of pregnancy, some chronic medical conditions (e.g., hypertension, diabetes, obesity, thrombotic vascular disease, systemic lupus erythematosus), some obstetric conditions (e.g., placental abruption, intrauterine growth restriction, fetal death), and conditions in which the uterus is rapidly enlarging (e.g., multiple gestations, polyhydramnios, hydatidiform mole). Although smoking is associated with more adverse pregnancy outcomes (e.g., ectopic pregnancies, placenta previa, placental abruption, FHR abnormalities, low birth weight, respiratory impairment in the newborn, and sudden infant death syndrome (SIDS)), there appears to be a 30% to 40% lower incidence of hypertensive disorders of pregnancy in women who smoke. The protective effect appears to be dose related, with heavy smokers having a lower frequency than those who smoke fewer cigarettes. In addition, women who are more physically active have a lower frequency of hypertensive disorders of pregnancy than sedentary women

77
Q
  1. Adverse effects (on the mother) associated with aortocaval compression by the gravid uterus include

A. Nausea and vomiting
B. Changes in cerebration
C. Fetal distress
D. All of the above

A

(D)Aortocaval compression, as its name suggests, produces both compression of the aorta (increase in afterload) as well as compression of the vena cava (decrease in venous return). The patient’s response is variable. Although some women have no symptoms, up to 15% of pregnant patients at term will, over several minutes in the supine position, develop hypotension and bradycardia (also called the supine hypotension syndrome). Some women will actually show an increase in brachial artery blood pressure due to the increase in afterload. These women may have a condition referred to as concealed hypotension (blood pressure above the compression that is adequate but blood pressure below the compression that is reduced). Because the blood supply to the uterus is distal to the aortic compression and uterine blood flow is decreased, the fetus may develop fetal distress. Other signs and symptoms of aortocaval compression include nausea, vomiting, pallor, sweating, and changes in cerebration. See also Question 701

78
Q
  1. Which of the following statements regarding a pregnant patient abusing cocaine is FALSE?

A. Hypertension, arrhythmias, myocardial ischemia, and tachycardia may occur with the rapid-sequence induction of general anesthesia in the acutely intoxicated patient
B. The minimum alveolar concentration (MAC) for general anesthetics is increased in chronic cocaine addicts
C. Some states consider in utero drug exposure to be a form of child abuse and require physicians to report these patients
D. If a vasopressor is needed to treat hypotension, phenylephrine is preferred over ephedrine

A

(B)Cocaine can produce life-threatening complications that are usually related to the accumulation of catecholamines, and patients may present with the classic signs of toxemia (i.e., hypertension and proteinuria) as well as chest pain. The typical half-life of cocaine is 30 to 90 minutes, but the acute effects can last as long as 6 hours. Because some states consider in utero cocaine exposure a form of child abuse that requires physicians to report positive drug tests in pregnant women, many patients who use cocaine have no prenatal care. Urine tests may be positive for 24 to 72 hours after cocaine use (depending on the amount used). Life-threatening events are more common with general than regional anesthesia. The most frequent problem with induction of general anesthesia is severe hypertension. Arrhythmias, myocardial ischemia, and tachycardia may also occur with the induction of general anesthesia. Labetalol and nitroglycerin have been used to treat these conditions. The MAC level is increased in patients who are acutely intoxicated, whereas patients chronically abusing cocaine have a lower MAC (due to the depletion of catecholamines). These patients are at risk for hypotension, which is commonly seen after the induction of regional anesthesia for cesarean section. Ephedrine may not be an effective vasopressor in these catecholamine-depleted patients. Phenylephrine, a direct-acting drug, is a better vasopressor

79
Q
  1. Each of the following is correct when advising the surgeon to perform infiltration anesthesia for an emergency cesarean delivery when general and neuraxial anesthesia are contraindicated EXCEPT

A. A midline incision is most desirable
B. The rectus muscle should be injected to provide good skin analgesia
C. Bupivacaine with bicarbonate is the local anesthetic of choice
D. Mild sedation with ketamine and midazolam is permissible

A

(C)In cases of emergency cesarean section when general anesthesia is contraindicated (e.g., poor airway when one questions one’s ability to intubate and/or ventilate the patient) and neuraxial anesthesia is contraindicated (e.g., severe hypovolemia or coagulopathy), emergency infiltration anesthesia is acceptable. All of the choices are correct except the choice of a local anesthetic. As the surgeon will be injecting a fair volume of local anesthetic (often 100mL), and as bupivacaine has a slow onset and potentially dangerous cardiac toxicity with large doses, bupivacaine is a poor choice. A dose of 0.5% lidocaine (plasma half-life of 90 minutes) is often used because it is readily available and relatively safe. Chloroprocaine may be safer because it also has a fast onset and its plasma half-life is extremely short (23 seconds). Both midazolam and ketamine may lead to some amnesia for the patient, which may be advantageous in this emergency situation; however, too much of the IV drugs could obtund the patient and may lead to aspiration of gastric contents. A good coach at the head of the bed may be invaluable for reassuring the patient as to the care

80
Q
  1. A 24-year-old primiparous woman is undergoing an elective cesarean section (breech position). After prehydration with 1500 mL of saline, a spinal anesthetic is performed; 5 minutes later, the blood pressure is noted to be 80/40 mm Hg and the heart rate is 110 beats/min. The BEST treatment (best fetal pH) after ensuring that adequate left uterine displacement is performed would be

A. Phenylephrine
B. Ephedrine
C. Epinephrine
D. 1000 mL 5% dextrose in lactated Ringer solution

A

(A)The most common complication after a spinal or epidural anesthetic is placed is systemic hypotension. Because the cardiac output is influenced by four main factors (preload, afterload, contractility, and heart rate and rhythm), treatment is directed at these factors. First, consider more left uterine displacement (which can increase venous return and preload). Next, administer more IV fluids (without dextrose such as lactated Ringer or saline solution) to increase preload if the amount of fluid used to prehydrate the patient before the block is thought to be inadequate. In this case the amount of prehydration with saline is an appropriate amount, but a little more non-dextrose containing fluid may be helpful. Intravenous fluids with dextrose are used only for maintenance fluids and should not be used to prevent or treat hypotension from regional anesthesia because the fluid load with dextrose causes significant maternal and fetal hyperglycemia and hyperinsulinemia. After delivery, the sugar supply for the newborn stops but the insulin response continues, often causing fetal hypoglycemia after delivery. It should be noted that 5% albumin solutions are expensive and are not recommended for routine use to treat hypotension. Vasopressors and/or drugs that increase cardiac contractility are commonly needed to increase afterload. Initial laboratory studies with pregnant ewes suggested that ephedrine was a better choice than phenylephrine or other α-adrenergic agonists when looking at changes in uterine blood flow. In these initial studies, the blood pressure was raised from normal to higher levels, and ephedrine was the drug of choice because phenylephrine decreased uterine blood flow, whereas ephedrine did not. However, raising a normal pressure to higher levels is not the same thing as raising a low blood pressure to normal. In more recent human studies looking at ephedrine and phenylephrine use, no difference was noted in the prophylactic or therapeutic use of these drugs for maternal hypotension. It was also noted that maternal bradycardia was more common with maternal phenylephrine administration, whereas maternal tachycardia was more common with maternal ephedrine administration; also, neonatal arterial pH was slightly higher when phenylephrine was used compared with ephedrine. Why this occurs is unclear but may be related to ephedrine’s ability to cross the placenta, causing β-adrenergic stimulation in the newborn (F/M blood ratio is 0.7 for ephedrine and 0.2 for phenylephrine). In this patient who has left uterine displacement, adequate IV hydration, and a heart rate of 110 beats/min, phenylephrine would be the preferred vasopressor. If the mother has hypotension with bradycardia, ephedrine might be a better choice. Epinephrine is rarely needed but should be available and used when there is severe hypotension that is not responsive to phenylephrine or ephedrine, especially when there is associated fetal bradycardia

81
Q
  1. A woman has been admitted for a dilation and evacuation (D&E) at 10 weeks’ EGA. She has some persistent bleeding and cramping after the expulsion of some tissue. Her obstetric condition is called

A. A threatened abortion
B. An inevitable abortion
C. A complete abortion
D. An incomplete abortion

A

(D)A threatened abortion is defined as uterine bleeding without cervical dilation before 20 weeks’ gestation. Bleeding may be accompanied by uterine cramps or backache. Half of these cases will go on to spontaneously abort. An inevitable abortion has cervical dilation and/or rupture of membranes and will spontaneously abort. A complete abortion occurs when there is complete expulsion of the fetus and the placenta, and in these cases there is no need for a dilation and curettage (D&C). If there is only partial expulsion of tissue, as in this case, an incomplete abortion has occurred, and this requires a D&E to remove the remaining fetal or placental tissue. In these cases the cervix has usually dilated some and the patient usually can be managed with some mild sedation, because the most painful part of a D&E is cervical dilation. A paracervical block can be most useful for pain control during the procedure if the cervix needs to be dilated. A fetal death that is unrecognized for several weeks is called a missed abortion, and if this occurs at an advanced gestational age, DIC may occasionally result. A habitual or recurrent abortion refers to the occurrence of three or more consecutive spontaneous abortions

82
Q
  1. Which of the following treatments has proven effective in decreasing the incidence of PDPHs after an accidental dural puncture with an epidural needle?

A. Bed rest
B. Prophylactic hydration
C. Prophylactic epidural blood patch after delivery
D. None of the above

A

(D)Accidental dural punctures with an epidural needle occur in about 1% of epidural attempts, and about a half of these patients develop a postdural puncture headache. Several techniques have been tried in an attempt to decrease the incidence of PDPHs but without success. Lying horizontally is helpful in relieving PDPHs, but prophylactic bed rest has not decreased the incidence of PDPH. Although avoidance of dehydration is recommended, excessive fluid intake has not been shown to decrease the incidence of PDPH. Epidural blood patches have proven successful in treating PDPH. However, the prophylactic injection of blood into the epidural space after the epidural block has worn off does not appear to significantly decrease the incidence of headaches but may decrease the duration of the headache. Keep in mind that the injection of blood through a contaminated epidural catheter may increase the risk of infection because blood is an excellent culture medium. For treatment, however, epidural blood patches are very successful, with better success rates if performed several hours after delivery compared with immediately after the delivery. Perhaps the leakage of cerebrospinal fluid into the epidural space dilutes the blood, making the patch weaker. Caffeine has been suggested by some studies to decrease the incidence of PDPHs, but other studies show no prophylactic benefit

83
Q
  1. Factors associated with advanced molar pregnancy (i.e., > 14 to 16-week size uterus) include all of the following EXCEPT

A. Hypertensive disorders of pregnancy
B. Hypothyroidism
C. Acute cardiopulmonary distress
D. Hyperemesis gravidarum

A

(B)Earlier diagnosis of complete molar pregnancies has decreased the incidence of medical complications. However, excessive uterine size occurs in up to one half of patients with a complete molar pregnancy and is associated with a high incidence of medical complications. Medical complications when the uterine size is greater than 14 to 16 weeks’ gestational size include ovarian theca-lutein cysts (4%-50%), hyperemesis gravidarum (15%-30%), hypertensive disorders of pregnancy (11%-27%), anemia (hemoglobin <10 g/dL) (10%-54%), acute cardiopulmonary distress (6%-27%), malignant sequelae (metastasis) (4%-36%), and hyperthyroidism (1%-7%)

84
Q
  1. Refractory cardiac arrest is MOST likely after the rapid unintentional IV injection of which of the following local anesthetics?

A. Lidocaine
B. Bupivacaine
C. Ropivacaine
D. Chloroprocaine

A

(B)Several cases of maternal cardiac arrest have occurred in pregnant women who were administered bupivacaine (Marcaine, Sensorcaine). Typically, the patients received an unintentional IV bolus of 0.75% bupivacaine intended for the epidural space. They had a brief grand mal seizure followed by cardiovascular collapse. Successful treatment was often prolonged and involved basic resuscitation (intubation, ventilation with 100% oxygen, cardiac compression with left uterine tilt, defibrillation, epinephrine, vasopressin, atropine), as well as rapid delivery of the fetus (if possible within 4-5 minutes). Delivery of the fetus makes successful resuscitation of the mother more likely. Incremental small injections of local anesthetic looking for toxicity should decrease the chance for cardiovascular collapse. Bupivacaine 0.75% now is considered contraindicated for use in the epidural space of parturients. Both levobupivacaine (Chirocaine) and ropivacaine (Naropin) were developed to have a long duration of action, like bupivacaine, but with less cardiac toxicity. Although these compounds have less cardiac toxicity than bupivacaine, they are more cardiac toxic than lidocaine (intermediate duration of action) and chloroprocaine (short duration of action)

85
Q
  1. American Society of Regional Anesthesia (ASRA) guidelines for the treatment of local anesthetic systemic toxicity (LAST) for cardiac arrhythmias include the use of Intralipid and the AVOIDANCE of all of the following drugs EXCEPT

A. Vasopressin
B. β-Blockers
C. Calcium channel blockers
D. Low-dose epinephrine (< 1 μg/kg)

A

(D)Treatment of the pregnant patient who develops cardiovascular collapse from LAST consists of positioning the patient with left lateral displacement and preparing for an emergency cesarean delivery. Consider delivery of the infant if the mother cannot be resuscitated within several minutes because delivery of the infant makes it easier to resuscitate the mother. Current (2017) ASRA guidelines for LAST for a patient with cardiovascular collapse include the following:
●Stop injecting the local anesthetic.
●Get help (Call for lipid emulsion, alert nearest cardiopulmonary bypass team because resuscitation may be prolonged).
●Manage airway (Ventilate with 100% oxygen, avoid hyperventilation, and place an advanced airway device if necessary).
●Control seizures (Benzodiazepines are preferred and avoid large doses of propofol, especially in hemodynamically unstable patients).
●Treat hypotension and bradycardia; if pulseless, start CPR.
●Start IV lipid emulsion (Intralipid 20%) therapy (initial bolus of Intralipid is 1.5mL/kg over 2-3 minutes, or about 100mL in an adult). The bolus dose is followed by a continuous infusion of 0.25mL/kg (ideal body weight)/hr. You can repeat the bolus dose one or two times for persistent cardiovascular collapse and then double the rate in a continuous infusion rate if the blood pressure remains low. Continue the infusion for at least 10 minutes after cardiovascular stability is attained. The upper limit of Intralipid (20%) is 10mL/kg over 30 minutes.
●Continue to monitor the patient for at least 4 to 6 hours after a cardiovascular event and at least 2 hours after a limited CNS event.
●AVOID the use of vasopressin, calcium channel blockers, β-blockers, and other local anesthetics.
●If you need epinephrine, the dose should be reduced to ≦1μg/kg epinephrine.
●Report LAST

86
Q
  1. Transient neurologic syndrome (TNS) is MOST commonly seen after the spinal anesthetic injection of which local anesthetic?

A. Lidocaine
B. Bupivacaine
C. Prilocaine
D. Tetracaine

A

(A)TNS, formerly called transient radicular irritation (TRI), occurs most commonly after spinal anesthesia with lidocaine (Xylocaine). Symptoms include back pain that develops after the block resolves and radiates to the buttocks and legs. The pain is not associated with motor or sensory loss or electromyographic changes. It can be severe, requiring hospital admission of outpatients, and typically resolves within 1 to 4 days. It appears to occur more commonly when outpatients are operated on in the lithotomy position and appears to be less likely when patients are pregnant

87
Q
  1. You have a well-working T10 labor epidural in a woman with a questionable difficult airway and have just been informed that an urgent cesarean section is needed for a nonreassuring FHR tracing. Which of the following local anesthetics would give you the SLOWEST onset of surgical anesthesia?

A. 3% chloroprocaine with freshly added epinephrine (1:200,000)
B. 2% lidocaine with freshly added epinephrine (1:200,000)
C. 2% lidocaine and epinephrine with added bicarbonate
D. 0.5% levobupivacaine with fentanyl

A

(D)When dealing with an urgent cesarean section in a patient with a well-functioning labor epidural, raising the level of anesthesia is often chosen. Of the commonly used local anesthetics, 3% 2-chloroprocaine with epinephrine 1:200,000 and 2% lidocaine with epinephrine 1:200,000 have much faster onsets of action than bupivacaine or levobupivacaine, which have relatively slow onsets of action. Because of the slow onset of action of bupivacaine or levobupivacaine, they are not recommended to raise an existing labor analgesic level for an urgent cesarean delivery. Alkalinization of the local anesthetic with bicarbonate shifts more of the local anesthetic molecules to the nonionized and more lipid-soluble form for a faster onset (and a more solid block); however, it does take a little time to mix the solution. Typically, 1mL of 8.4% sodium bicarbonate (1mEq/mL) is added to each 10mL of 3% 2-chloroprocaine with epinephrine 1:200,000 or 2% lidocaine with epinephrine 1:200,000, and those solutions in a dose of 15 to 20mL can usually raise a labor epidural to cesarean levels in <5 minutes. If you do not add the bicarbonate to 2% lidocaine with epinephrine 1:200,000, the onset time is twice as long, or about 10 minutes. Fentanyl can be added (75-100 μg) to the local anesthetic solution for a more solid block and for some postoperative analgesia

88
Q
  1. Which local anesthetic has the SHORTEST plasma half-life after being injected into the epidural space?

A. Bupivacaine
B. Chloroprocaine
C. Lidocaine
D. Ropivacaine

A

(B)The blood level of a local anesthetic is based on the absorption of the local anesthetic into the bloodstream and on the local anesthetic’s metabolism. Ester local anesthetics (e.g., procaine, chloroprocaine, and tetracaine) undergo metabolism by pseudocholinesterase and other plasma esterases, whereas amide local anesthetics (e.g., lidocaine, mepivacaine, prilocaine, bupivacaine, levobupivacaine, ropivacaine, and etidocaine) are not metabolized in the bloodstream but require liver metabolism and have much longer half-lives. If chloroprocaine (Nesacaine) is injected directly into plasma, the in vitro half-life is 11 to 21 seconds in maternal plasma and 43 seconds in fetal plasma. After an epidural injection, where there is absorption of local anesthetic from the injection site as well as metabolism, the maternal half-life for chloroprocaine is <7 minutes, for lidocaine is 110 to 120 minutes, for ropivacaine is 5 to 6 hours, and for bupivacaine is 10 to 12 hours